Rocksolid Light

Welcome to novaBBS (click a section below)

mail  files  register  newsreader  groups  login

Message-ID:  

We can predict everything, except the future.


tech / sci.physics.relativity / Re: The De Sitter Challenge - Has to be answered

SubjectAuthor
* The De Sitter Challenge - Has to be answeredgehan.am...@gmail.com
+* Re: The De Sitter Challenge - Has to be answeredSylvia Else
|`* Re: The De Sitter Challenge - Has to be answeredgehan.am...@gmail.com
| `* Re: The De Sitter Challenge - Has to be answeredSylvia Else
|  `* Re: The De Sitter Challenge - Has to be answeredgehan.am...@gmail.com
|   +* Re: The De Sitter Challenge - Has to be answeredpatdolan
|   |`- Re: The De Sitter Challenge - Has to be answeredgehan.am...@gmail.com
|   `* Re: The De Sitter Challenge - Has to be answeredTrevor Lange
|    `* Re: The De Sitter Challenge - Has to be answeredgehan.am...@gmail.com
|     +- Re: The De Sitter Challenge - Has to be answeredSylvia Else
|     `* Re: The De Sitter Challenge - Has to be answeredTrevor Lange
|      `* Re: The De Sitter Challenge - Has to be answeredgehan.am...@gmail.com
|       +* Re: The De Sitter Challenge - Has to be answeredSylvia Else
|       |`* Re: The De Sitter Challenge - Has to be answeredgehan.am...@gmail.com
|       | `* Re: The De Sitter Challenge - Has to be answeredSylvia Else
|       |  +* Re: The De Sitter Challenge - Has to be answeredgehan.am...@gmail.com
|       |  |+* Re: The De Sitter Challenge - Has to be answeredDono.
|       |  ||`* Re: The De Sitter Challenge - Has to be answeredgehan.am...@gmail.com
|       |  || `- Re: The De Sitter Challenge - Has to be answeredDono.
|       |  |`* Re: The De Sitter Challenge - Has to be answeredSylvia Else
|       |  | `* Re: The De Sitter Challenge - Has to be answeredgehan.am...@gmail.com
|       |  |  `- Re: The De Sitter Challenge - Has to be answeredSylvia Else
|       |  `- Re: The De Sitter Challenge - Has to be answeredgehan.am...@gmail.com
|       +* Re: The De Sitter Challenge - Has to be answeredTrevor Lange
|       |`* Re: The De Sitter Challenge - Has to be answeredgehan.am...@gmail.com
|       | `* Re: The De Sitter Challenge - Has to be answeredTrevor Lange
|       |  `* Re: The De Sitter Challenge - Has to be answeredgehan.am...@gmail.com
|       |   `* Re: The De Sitter Challenge - Has to be answeredTrevor Lange
|       |    `* Re: The De Sitter Challenge - Has to be answeredgehan.am...@gmail.com
|       |     `* Re: The De Sitter Challenge - Has to be answeredTrevor Lange
|       |      +* Re: The De Sitter Challenge - Has to be answeredgehan.am...@gmail.com
|       |      |`* Re: The De Sitter Challenge - Has to be answeredTrevor Lange
|       |      | +* Re: The De Sitter Challenge - Has to be answeredgehan.am...@gmail.com
|       |      | |`- Re: The De Sitter Challenge - Has to be answeredTom Roberts
|       |      | +- Re: The De Sitter Challenge - Has to be answeredTrevor Lange
|       |      | +- Re: The De Sitter Challenge - Has to be answeredgehan.am...@gmail.com
|       |      | +- Re: The De Sitter Challenge - Has to be answeredTrevor Lange
|       |      | +- Re: The De Sitter Challenge - Has to be answeredgehan.am...@gmail.com
|       |      | +- Re: The De Sitter Challenge - Has to be answeredTrevor Lange
|       |      | +- Re: The De Sitter Challenge - Has to be answeredgehan.am...@gmail.com
|       |      | +- Re: The De Sitter Challenge - Has to be answeredTrevor Lange
|       |      | +- Re: The De Sitter Challenge - Has to be answeredgehan.am...@gmail.com
|       |      | +- Re: The De Sitter Challenge - Has to be answeredTrevor Lange
|       |      | +- Re: The De Sitter Challenge - Has to be answeredgehan.am...@gmail.com
|       |      | +- Re: The De Sitter Challenge - Has to be answeredTrevor Lange
|       |      | +- Re: The De Sitter Challenge - Has to be answeredgehan.am...@gmail.com
|       |      | +- Re: The De Sitter Challenge - Has to be answeredPaparios
|       |      | +- Re: The De Sitter Challenge - Has to be answeredMaciej Wozniak
|       |      | +- Re: The De Sitter Challenge - Has to be answeredTrevor Lange
|       |      | +- Re: The De Sitter Challenge - Has to be answeredTrevor Lange
|       |      | +- Re: The De Sitter Challenge - Has to be answeredTrevor Lange
|       |      | +- Re: The De Sitter Challenge - Has to be answeredgehan.am...@gmail.com
|       |      | +- Re: The De Sitter Challenge - Has to be answeredgehan.am...@gmail.com
|       |      | +- Re: The De Sitter Challenge - Has to be answeredgehan.am...@gmail.com
|       |      | +- Re: The De Sitter Challenge - Has to be answeredgehan.am...@gmail.com
|       |      | +- Re: The De Sitter Challenge - Has to be answeredTrevor Lange
|       |      | +- Re: The De Sitter Challenge - Has to be answeredPaparios
|       |      | +- Re: The De Sitter Challenge - Has to be answeredRichD
|       |      | `- Re: The De Sitter Challenge - Has to be answeredProkaryotic Capase Homolog
|       |      `* Re: The De Sitter Challenge - Has to be answeredgehan.am...@gmail.com
|       |       +* Re: The De Sitter Challenge - Has to be answeredJane
|       |       |+* Re: The De Sitter Challenge - Has to be answeredTom Roberts
|       |       ||`* Re: The De Sitter Challenge - Has to be answeredJane
|       |       || `* Re: The De Sitter Challenge - Has to be answeredPaul Alsing
|       |       ||  +- Re: The De Sitter Challenge - Has to be answeredJane
|       |       ||  `* Re: The De Sitter Challenge - Has to be answeredgehan.am...@gmail.com
|       |       ||   +* Re: The De Sitter Challenge - Has to be answeredVolney
|       |       ||   |+- Re: The De Sitter Challenge - Has to be answeredTom Roberts
|       |       ||   |`- Re: The De Sitter Challenge - Has to be answeredDono.
|       |       ||   `* Re: The De Sitter Challenge - Has to be answeredgehan.am...@gmail.com
|       |       ||    `* Re: The De Sitter Challenge - Has to be answeredVolney
|       |       ||     `- Re: The De Sitter Challenge - Has to be answeredRoss Finlayson
|       |       |`* Re: The De Sitter Challenge - Has to be answeredLaurence Clark Crossen
|       |       | +- Re: The De Sitter Challenge - Has to be answeredJ. J. Lodder
|       |       | +* Re: The De Sitter Challenge - Has to be answeredTom Roberts
|       |       | |`- Re: The De Sitter Challenge - Has to be answeredJane
|       |       | +- Re: The De Sitter Challenge - Has to be answeredMaciej Wozniak
|       |       | `- Re: The De Sitter Challenge - Has to be answeredLaurence Clark Crossen
|       |       `* Re: The De Sitter Challenge - Has to be answeredMikko
|       |        +- Re: The De Sitter Challenge - Has to be answeredgehan.am...@gmail.com
|       |        `* Re: The De Sitter Challenge - Has to be answeredwhodat
|       |         `* Re: The De Sitter Challenge - Has to be answeredWeslee Alberici
|       |          `- Re: The De Sitter Challenge - Has to be answeredwhodat
|       `* Re: The De Sitter Challenge - Has to be answeredProkaryotic Capase Homolog
|        `- Re: The De Sitter Challenge - Has to be answeredgehan.am...@gmail.com
+* Re: The De Sitter Challenge - Has to be answeredPaul B. Andersen
|`- Re: The De Sitter Challenge - Has to be answeredgehan.am...@gmail.com
+* Re: The De Sitter Challenge - Has to be answeredTom Roberts
|`- Re: The De Sitter Challenge - Has to be answeredMaciej Wozniak
`- Re: The De Sitter Challenge - Has to be answeredRoss Finlayson

Pages:1234
Re: The De Sitter Challenge - Has to be answered

<YPadnfL9vunQfMj5nZ2dnZfqlJxh4p2d@giganews.com>

  copy mid

https://www.novabbs.com/tech/article-flat.php?id=114750&group=sci.physics.relativity#114750

  copy link   Newsgroups: sci.physics.relativity
Path: i2pn2.org!i2pn.org!usenet.blueworldhosting.com!diablo1.usenet.blueworldhosting.com!peer02.iad!feed-me.highwinds-media.com!news.highwinds-media.com!border-1.nntp.ord.giganews.com!nntp.giganews.com!Xl.tags.giganews.com!local-2.nntp.ord.giganews.com!news.giganews.com.POSTED!not-for-mail
NNTP-Posting-Date: Sat, 06 May 2023 05:10:37 +0000
Date: Sat, 6 May 2023 00:10:37 -0500
MIME-Version: 1.0
User-Agent: Mozilla/5.0 (Macintosh; Intel Mac OS X 10.15; rv:102.0)
Gecko/20100101 Thunderbird/102.10.1
Subject: Re: The De Sitter Challenge - Has to be answered
Newsgroups: sci.physics.relativity
References: <f8985602-5407-41f0-ae98-d2f9822bafaan@googlegroups.com>
<217730f2-8df8-444a-9dc8-91ff22fe8e0an@googlegroups.com>
<kb9rquF9c3eU1@mid.individual.net>
<d8cf070e-eecc-4d6a-8b07-761033b39d82n@googlegroups.com>
<14ab18f7-e73a-4669-9711-d948e9eb9a03n@googlegroups.com>
<47b7e21e-3092-48f1-b4c8-ce7b7c9c95ebn@googlegroups.com>
<db6709a7-14c2-44be-a858-ddff3edb5953n@googlegroups.com>
<ba2a0cf6-8427-47a1-9362-af508819f5c1n@googlegroups.com>
<9af61d5f-1ba5-4d8a-b74a-8eb8af2240adn@googlegroups.com>
<836c5b12-c66c-4264-9ce5-8485c1292dcdn@googlegroups.com>
<c2fe1ba0-e01b-4c6a-b7ff-149d4f9045c4n@googlegroups.com>
<ee14994b-3112-47d7-8a03-551c3f636987n@googlegroups.com>
<55cf99e6-1675-4d83-b0b3-f5fed4cf6911n@googlegroups.com>
<5c054fd2-88fc-43da-9bc7-1ea0aa279336n@googlegroups.com>
<abc12fce-3f84-4b4a-92fb-a715c2115d5en@googlegroups.com>
<f077e8f9-2432-46c9-a761-82f5fb925d1dn@googlegroups.com>
<175c164593ff4b9e$14$406094$4bd3c1de@news.newsgroupdirect.com>
Content-Language: en-US
From: tjoberts...@sbcglobal.net (Tom Roberts)
In-Reply-To: <175c164593ff4b9e$14$406094$4bd3c1de@news.newsgroupdirect.com>
Content-Type: text/plain; charset=UTF-8; format=flowed
Content-Transfer-Encoding: 7bit
Message-ID: <YPadnfL9vunQfMj5nZ2dnZfqlJxh4p2d@giganews.com>
Lines: 12
X-Usenet-Provider: http://www.giganews.com
X-Trace: sv3-CmbJsyMX4uE2pUg5ElAMjnG+gqZaG5wPweEmy4kcv0uqHfL7ukbyR5P/IVlGe7YbnTK+YZtksdnWFb5!CT4dso7WGeFiwGKFb9cwYnLhlGAkq30o4UvVVtogiwBhseOSMiKkZJcXyasN3RUw/dQOIoNfCA==
X-Complaints-To: abuse@giganews.com
X-DMCA-Notifications: http://www.giganews.com/info/dmca.html
X-Abuse-and-DMCA-Info: Please be sure to forward a copy of ALL headers
X-Abuse-and-DMCA-Info: Otherwise we will be unable to process your complaint properly
X-Postfilter: 1.3.40
X-Received-Bytes: 2720
 by: Tom Roberts - Sat, 6 May 2023 05:10 UTC

On 5/4/23 7:00 PM, Jane wrote:
> Now here is NOW everywhere even if there is no way to actually prove
> that.

Once again you display your personal ignorance. That claim takes you out
of the realm of science and into theology -- your wishes and dreams are
no constraint on the world we inhabit, or on the theories we construct
to describe that world. They do, however, seem to be a constraint on how
you "think"....

Tom Roberts

Re: The De Sitter Challenge - Has to be answered

<YPadne39vulWfMj5nZ2dnZfqlJz8fwAA@giganews.com>

  copy mid

https://www.novabbs.com/tech/article-flat.php?id=114751&group=sci.physics.relativity#114751

  copy link   Newsgroups: sci.physics.relativity
Path: i2pn2.org!i2pn.org!weretis.net!feeder6.news.weretis.net!news.misty.com!border-2.nntp.ord.giganews.com!nntp.giganews.com!Xl.tags.giganews.com!local-1.nntp.ord.giganews.com!news.giganews.com.POSTED!not-for-mail
NNTP-Posting-Date: Sat, 06 May 2023 05:12:43 +0000
Date: Sat, 6 May 2023 00:12:43 -0500
MIME-Version: 1.0
User-Agent: Mozilla/5.0 (Macintosh; Intel Mac OS X 10.15; rv:102.0)
Gecko/20100101 Thunderbird/102.10.1
Subject: Re: The De Sitter Challenge - Has to be answered
Content-Language: en-US
Newsgroups: sci.physics.relativity
References: <f8985602-5407-41f0-ae98-d2f9822bafaan@googlegroups.com>
<kb9rquF9c3eU1@mid.individual.net>
<d8cf070e-eecc-4d6a-8b07-761033b39d82n@googlegroups.com>
<14ab18f7-e73a-4669-9711-d948e9eb9a03n@googlegroups.com>
<47b7e21e-3092-48f1-b4c8-ce7b7c9c95ebn@googlegroups.com>
<db6709a7-14c2-44be-a858-ddff3edb5953n@googlegroups.com>
<ba2a0cf6-8427-47a1-9362-af508819f5c1n@googlegroups.com>
<9af61d5f-1ba5-4d8a-b74a-8eb8af2240adn@googlegroups.com>
<836c5b12-c66c-4264-9ce5-8485c1292dcdn@googlegroups.com>
<c2fe1ba0-e01b-4c6a-b7ff-149d4f9045c4n@googlegroups.com>
<ee14994b-3112-47d7-8a03-551c3f636987n@googlegroups.com>
<55cf99e6-1675-4d83-b0b3-f5fed4cf6911n@googlegroups.com>
<5c054fd2-88fc-43da-9bc7-1ea0aa279336n@googlegroups.com>
<abc12fce-3f84-4b4a-92fb-a715c2115d5en@googlegroups.com>
<fd462aa7-b6b3-42d1-8f81-22b9326b4b8en@googlegroups.com>
<cf311410-2bb9-4a4e-b0b7-647f3a88230fn@googlegroups.com>
<3457c613-1a84-4676-826f-a3fa0ab2c0fbn@googlegroups.com>
From: tjoberts...@sbcglobal.net (Tom Roberts)
In-Reply-To: <3457c613-1a84-4676-826f-a3fa0ab2c0fbn@googlegroups.com>
Content-Type: text/plain; charset=UTF-8; format=flowed
Content-Transfer-Encoding: 7bit
Message-ID: <YPadne39vulWfMj5nZ2dnZfqlJz8fwAA@giganews.com>
Lines: 9
X-Usenet-Provider: http://www.giganews.com
X-Trace: sv3-nAj13xzB2cTLfhKpDh6QiRm6LDjj0o0EZ2BXdJVSNacfQlqEtu8jzRR4alkfC4W3V1K9bhHTqOPF0dJ!4VwfxXO31iBYVNAmm49pE84AdW/Ory8b9k/LG3I0mcuPV0Ei9V+LSuInIF9ZqU5gGcUElw5vJw==
X-Complaints-To: abuse@giganews.com
X-DMCA-Notifications: http://www.giganews.com/info/dmca.html
X-Abuse-and-DMCA-Info: Please be sure to forward a copy of ALL headers
X-Abuse-and-DMCA-Info: Otherwise we will be unable to process your complaint properly
X-Postfilter: 1.3.40
 by: Tom Roberts - Sat, 6 May 2023 05:12 UTC

On 5/4/23 10:26 PM, gehan.am...@gmail.com wrote:
> is it possible to come up with a logical disproof of the second
> postulate, that is, a set steps that will lead to a contradiction?

That is impossible -- armchair fantasies and gedanken experiments have
no ability to refute SR or GR -- that requires a real experiment, and to
date that has not happened.

Tom Roberts

Re: The De Sitter Challenge - Has to be answered

<d760773b-7579-4901-95d6-d5d1b4bbd71cn@googlegroups.com>

  copy mid

https://www.novabbs.com/tech/article-flat.php?id=114754&group=sci.physics.relativity#114754

  copy link   Newsgroups: sci.physics.relativity
X-Received: by 2002:a05:620a:3715:b0:74a:dd04:6146 with SMTP id de21-20020a05620a371500b0074add046146mr1168087qkb.3.1683352204616;
Fri, 05 May 2023 22:50:04 -0700 (PDT)
X-Received: by 2002:a05:6214:9c9:b0:5e6:5f7b:d1a3 with SMTP id
dp9-20020a05621409c900b005e65f7bd1a3mr570034qvb.2.1683352204448; Fri, 05 May
2023 22:50:04 -0700 (PDT)
Path: i2pn2.org!i2pn.org!usenet.goja.nl.eu.org!3.eu.feeder.erje.net!feeder.erje.net!border-1.nntp.ord.giganews.com!nntp.giganews.com!news-out.google.com!nntp.google.com!postnews.google.com!google-groups.googlegroups.com!not-for-mail
Newsgroups: sci.physics.relativity
Date: Fri, 5 May 2023 22:50:04 -0700 (PDT)
In-Reply-To: <dc083541-a0c2-4e83-ba38-dee0e7036778n@googlegroups.com>
Injection-Info: google-groups.googlegroups.com; posting-host=2601:601:1700:7df0:5155:93ae:8d2f:fb3b;
posting-account=B2MNBQoAAADtgq_pZTEECSkLIDJGrDSJ
NNTP-Posting-Host: 2601:601:1700:7df0:5155:93ae:8d2f:fb3b
References: <f8985602-5407-41f0-ae98-d2f9822bafaan@googlegroups.com>
<kb9fh2F7st1U1@mid.individual.net> <217730f2-8df8-444a-9dc8-91ff22fe8e0an@googlegroups.com>
<kb9rquF9c3eU1@mid.individual.net> <d8cf070e-eecc-4d6a-8b07-761033b39d82n@googlegroups.com>
<14ab18f7-e73a-4669-9711-d948e9eb9a03n@googlegroups.com> <47b7e21e-3092-48f1-b4c8-ce7b7c9c95ebn@googlegroups.com>
<db6709a7-14c2-44be-a858-ddff3edb5953n@googlegroups.com> <ba2a0cf6-8427-47a1-9362-af508819f5c1n@googlegroups.com>
<9af61d5f-1ba5-4d8a-b74a-8eb8af2240adn@googlegroups.com> <836c5b12-c66c-4264-9ce5-8485c1292dcdn@googlegroups.com>
<c2fe1ba0-e01b-4c6a-b7ff-149d4f9045c4n@googlegroups.com> <ee14994b-3112-47d7-8a03-551c3f636987n@googlegroups.com>
<55cf99e6-1675-4d83-b0b3-f5fed4cf6911n@googlegroups.com> <5c054fd2-88fc-43da-9bc7-1ea0aa279336n@googlegroups.com>
<abc12fce-3f84-4b4a-92fb-a715c2115d5en@googlegroups.com> <fd462aa7-b6b3-42d1-8f81-22b9326b4b8en@googlegroups.com>
<cf311410-2bb9-4a4e-b0b7-647f3a88230fn@googlegroups.com> <3457c613-1a84-4676-826f-a3fa0ab2c0fbn@googlegroups.com>
<f4c20a5f-8e0a-4db7-b955-56c97c17998bn@googlegroups.com> <de55a948-dd8d-4a0b-9025-8c3c491e8cd0n@googlegroups.com>
<64961caf-876c-4fc6-b7fd-f4903bbe0733n@googlegroups.com> <dc083541-a0c2-4e83-ba38-dee0e7036778n@googlegroups.com>
User-Agent: G2/1.0
MIME-Version: 1.0
Message-ID: <d760773b-7579-4901-95d6-d5d1b4bbd71cn@googlegroups.com>
Subject: Re: The De Sitter Challenge - Has to be answered
From: trevorla...@gmail.com (Trevor Lange)
Injection-Date: Sat, 06 May 2023 05:50:04 +0000
Content-Type: text/plain; charset="UTF-8"
Content-Transfer-Encoding: quoted-printable
Lines: 67
 by: Trevor Lange - Sat, 6 May 2023 05:50 UTC

On Friday, May 5, 2023 at 10:09:36 PM UTC-7, gehan.am...@gmail.com wrote:
> > Why would you think about disproving something that you understand to be true?
>
> Right now I am entertaining the thought not accepting it

That's dishonest, because you evaded facing the falsification of your beliefs by claiming that you understood the facts, but now you admit that you did not understand the facts, you were merely lying and evading. So if you have any intellectual integrity, you are now obligated to go back and honestly face the falsification of your beliefs. Understand?

> as it is not required.

Again, you previously lied and said you understood why the inertia of energy requires local Lorentz invariance, but now you admit your lie, so you need to go back and honestly confront the falsification of your beliefs.

> In the train and lightning strikes thought experiment, it is possible to say which
> lightning strike happened before the other in by conducting an investigation after
> the fact.

It is easy to determine all the objective facts involving the strike events A and B, such as the fact that tA < tB and t'A > t'B for relatively moving systems of standard inertial coordinates x,t and x',t'. Do you understand this? (Remember, I'm not asking if you entertain this, I'm asking if you understand this. Please answer honestly.)

> I do not need to assume anything is true, just that it works.

There is no need to assume things (in the sense you mean), the objective facts are all easily ascertainable and understandable.

> I need to read more that is all.

No, you need to *think* more, and be more honest. You are already in possession of all the information needed to understand special relativity. All you need to do is think rationally.

> "Because signals and other causal influences cannot travel faster than light "
> Simply stating the assumption.

Again, velocities can only be defined in terms of coordinate systems, and it is not an assumption but an empirical fact that light propagates in vacuum at speed c in terms of every standard system of inertial coordinates. Do you understand this?

> "you should use your own brain and your own understanding."
> As far as I know that is what I am doing.

No, that is not what you are doing. You are carefully and dishonestly evading all rational thought. Your pattern of textual misconstrual and passive-aggressive evasion has evidently been evolved and refined over a period of many years, so by now it is second nature to you, and you mistake it for thinking.

> Pythogoras' theorem can be easily demonstrated to be true visually.
> With Special Relativity the background assumptions are not so easily
> discerned.

But it's been fully and clearly explained to you, and the clear demonstrations have been pointed out to you, and it isn't all that difficult (e.g., comparable to Euclidean geometry), you simply run away from all rational engagement.

Re: The De Sitter Challenge - Has to be answered

<d5e0aeb2-cba6-4d84-a72c-506a1e4c7164n@googlegroups.com>

  copy mid

https://www.novabbs.com/tech/article-flat.php?id=114758&group=sci.physics.relativity#114758

  copy link   Newsgroups: sci.physics.relativity
X-Received: by 2002:a05:622a:1a89:b0:3e3:7dd2:47fc with SMTP id s9-20020a05622a1a8900b003e37dd247fcmr1169824qtc.10.1683355342220;
Fri, 05 May 2023 23:42:22 -0700 (PDT)
X-Received: by 2002:ad4:4d51:0:b0:61b:5f52:6867 with SMTP id
m17-20020ad44d51000000b0061b5f526867mr603955qvm.7.1683355341942; Fri, 05 May
2023 23:42:21 -0700 (PDT)
Path: i2pn2.org!i2pn.org!weretis.net!feeder6.news.weretis.net!news.misty.com!border-2.nntp.ord.giganews.com!nntp.giganews.com!news-out.google.com!nntp.google.com!postnews.google.com!google-groups.googlegroups.com!not-for-mail
Newsgroups: sci.physics.relativity
Date: Fri, 5 May 2023 23:42:21 -0700 (PDT)
In-Reply-To: <d760773b-7579-4901-95d6-d5d1b4bbd71cn@googlegroups.com>
Injection-Info: google-groups.googlegroups.com; posting-host=185.215.35.226; posting-account=sVBCDQoAAAADe-Ogi2R38m91EmLrcIgt
NNTP-Posting-Host: 185.215.35.226
References: <f8985602-5407-41f0-ae98-d2f9822bafaan@googlegroups.com>
<kb9fh2F7st1U1@mid.individual.net> <217730f2-8df8-444a-9dc8-91ff22fe8e0an@googlegroups.com>
<kb9rquF9c3eU1@mid.individual.net> <d8cf070e-eecc-4d6a-8b07-761033b39d82n@googlegroups.com>
<14ab18f7-e73a-4669-9711-d948e9eb9a03n@googlegroups.com> <47b7e21e-3092-48f1-b4c8-ce7b7c9c95ebn@googlegroups.com>
<db6709a7-14c2-44be-a858-ddff3edb5953n@googlegroups.com> <ba2a0cf6-8427-47a1-9362-af508819f5c1n@googlegroups.com>
<9af61d5f-1ba5-4d8a-b74a-8eb8af2240adn@googlegroups.com> <836c5b12-c66c-4264-9ce5-8485c1292dcdn@googlegroups.com>
<c2fe1ba0-e01b-4c6a-b7ff-149d4f9045c4n@googlegroups.com> <ee14994b-3112-47d7-8a03-551c3f636987n@googlegroups.com>
<55cf99e6-1675-4d83-b0b3-f5fed4cf6911n@googlegroups.com> <5c054fd2-88fc-43da-9bc7-1ea0aa279336n@googlegroups.com>
<abc12fce-3f84-4b4a-92fb-a715c2115d5en@googlegroups.com> <fd462aa7-b6b3-42d1-8f81-22b9326b4b8en@googlegroups.com>
<cf311410-2bb9-4a4e-b0b7-647f3a88230fn@googlegroups.com> <3457c613-1a84-4676-826f-a3fa0ab2c0fbn@googlegroups.com>
<f4c20a5f-8e0a-4db7-b955-56c97c17998bn@googlegroups.com> <de55a948-dd8d-4a0b-9025-8c3c491e8cd0n@googlegroups.com>
<64961caf-876c-4fc6-b7fd-f4903bbe0733n@googlegroups.com> <dc083541-a0c2-4e83-ba38-dee0e7036778n@googlegroups.com>
<d760773b-7579-4901-95d6-d5d1b4bbd71cn@googlegroups.com>
User-Agent: G2/1.0
MIME-Version: 1.0
Message-ID: <d5e0aeb2-cba6-4d84-a72c-506a1e4c7164n@googlegroups.com>
Subject: Re: The De Sitter Challenge - Has to be answered
From: gehan.am...@gmail.com (gehan.am...@gmail.com)
Injection-Date: Sat, 06 May 2023 06:42:22 +0000
Content-Type: text/plain; charset="UTF-8"
Content-Transfer-Encoding: quoted-printable
Lines: 93
 by: gehan.am...@gmail.co - Sat, 6 May 2023 06:42 UTC

On Saturday, May 6, 2023 at 10:50:05 AM UTC+5, Trevor Lange wrote:
> On Friday, May 5, 2023 at 10:09:36 PM UTC-7, gehan.am...@gmail.com wrote:
> > > Why would you think about disproving something that you understand to be true?
> >
> > Right now I am entertaining the thought not accepting it
> That's dishonest, because you evaded facing the falsification of your beliefs by claiming that you understood the facts, but now you admit that you did not understand the facts, you were merely lying and evading. So if you have any intellectual integrity, you are now obligated to go back and honestly face the falsification of your beliefs. Understand?

It is possible to understand a portrayal of reality without accepting it as fact. It happens all the time especially in religion.

> > as it is not required.
> Again, you previously lied and said you understood why the inertia of energy requires local Lorentz invariance, but now you admit your lie, so you need to go back and honestly confront the falsification of your beliefs.

I do understand the need for Lorentz invariance, not about inertial of energy.

> > In the train and lightning strikes thought experiment, it is possible to say which
> > lightning strike happened before the other in by conducting an investigation after
> > the fact.
> It is easy to determine all the objective facts involving the strike events A and B, such as the fact that tA < tB and t'A > t'B for relatively moving systems of standard inertial coordinates x,t and x',t'. Do you understand this? (Remember, I'm not asking if you entertain this, I'm asking if you understand this. Please answer honestly.)

No. I understand that any event can be translated into another coordinate system.

> > I do not need to assume anything is true, just that it works.
> There is no need to assume things (in the sense you mean), the objective facts are all easily ascertainable and understandable.
> > I need to read more that is all.
> No, you need to *think* more, and be more honest. You are already in possession of all the information needed to understand special relativity. All you need to do is think rationally.

I am thinking rationally as far as I am aware and that is true.

> > "Because signals and other causal influences cannot travel faster than light "
> > Simply stating the assumption.
> Again, velocities can only be defined in terms of coordinate systems, and it is not an assumption but an empirical fact that light propagates in vacuum at speed c in terms of every standard system of inertial coordinates. Do you understand this?
> > "you should use your own brain and your own understanding."
> > As far as I know that is what I am doing.

I do not accept that "that light propagates in vacuum at speed c in terms of every standard system of inertial coordinates. "
I do not believe it, I do not think it is true.

Now what?

Being in error is not mutually exclusive from using ones own brain and own understanding. These may be flawed.
Call it flawed then.

> No, that is not what you are doing. You are carefully and dishonestly evading all rational thought. Your pattern of textual misconstrual and passive-aggressive evasion has evidently been evolved and refined over a period of many years, so by now it is second nature to you, and you mistake it for thinking.
> > Pythogoras' theorem can be easily demonstrated to be true visually.
> > With Special Relativity the background assumptions are not so easily
> > discerned.
> But it's been fully and clearly explained to you, and the clear demonstrations have been pointed out to you, and it isn't all that difficult (e.g., comparable to Euclidean geometry), you simply run away from all rational engagement.

What I know to be rationality, or my practice of rationality does not allow me to accept that light is measured by all 'observers'as travelling at c.

So that is a problem with my rationality, then I am irrational, but according to you, since I can be rational and accept by irrationality nor can I be irrational and accept that I am irrational. I can also be ignorant, just like Dr. Albert Einstein was unaware of the nature of things , so they say, before 1900.

The De Sitter discussion can now be considered closed, I will retire to the books and pen and paper and the recommended books to look at this further. I expect to be back in a few weeks.

Re: The De Sitter Challenge - Has to be answered

<291e7827-e487-4b50-8c22-c127d0607d18n@googlegroups.com>

  copy mid

https://www.novabbs.com/tech/article-flat.php?id=114817&group=sci.physics.relativity#114817

  copy link   Newsgroups: sci.physics.relativity
X-Received: by 2002:ad4:550c:0:b0:61b:6026:5f34 with SMTP id pz12-20020ad4550c000000b0061b60265f34mr1033392qvb.2.1683399431975;
Sat, 06 May 2023 11:57:11 -0700 (PDT)
X-Received: by 2002:ac8:5709:0:b0:3e4:e5bf:a24f with SMTP id
9-20020ac85709000000b003e4e5bfa24fmr1809365qtw.7.1683399431730; Sat, 06 May
2023 11:57:11 -0700 (PDT)
Path: i2pn2.org!i2pn.org!usenet.blueworldhosting.com!diablo1.usenet.blueworldhosting.com!peer03.iad!feed-me.highwinds-media.com!news.highwinds-media.com!news-out.google.com!nntp.google.com!postnews.google.com!google-groups.googlegroups.com!not-for-mail
Newsgroups: sci.physics.relativity
Date: Sat, 6 May 2023 11:57:11 -0700 (PDT)
In-Reply-To: <d5e0aeb2-cba6-4d84-a72c-506a1e4c7164n@googlegroups.com>
Injection-Info: google-groups.googlegroups.com; posting-host=2601:601:1700:7df0:9a5:bb7b:2284:6c85;
posting-account=B2MNBQoAAADtgq_pZTEECSkLIDJGrDSJ
NNTP-Posting-Host: 2601:601:1700:7df0:9a5:bb7b:2284:6c85
References: <f8985602-5407-41f0-ae98-d2f9822bafaan@googlegroups.com>
<kb9fh2F7st1U1@mid.individual.net> <217730f2-8df8-444a-9dc8-91ff22fe8e0an@googlegroups.com>
<kb9rquF9c3eU1@mid.individual.net> <d8cf070e-eecc-4d6a-8b07-761033b39d82n@googlegroups.com>
<14ab18f7-e73a-4669-9711-d948e9eb9a03n@googlegroups.com> <47b7e21e-3092-48f1-b4c8-ce7b7c9c95ebn@googlegroups.com>
<db6709a7-14c2-44be-a858-ddff3edb5953n@googlegroups.com> <ba2a0cf6-8427-47a1-9362-af508819f5c1n@googlegroups.com>
<9af61d5f-1ba5-4d8a-b74a-8eb8af2240adn@googlegroups.com> <836c5b12-c66c-4264-9ce5-8485c1292dcdn@googlegroups.com>
<c2fe1ba0-e01b-4c6a-b7ff-149d4f9045c4n@googlegroups.com> <ee14994b-3112-47d7-8a03-551c3f636987n@googlegroups.com>
<55cf99e6-1675-4d83-b0b3-f5fed4cf6911n@googlegroups.com> <5c054fd2-88fc-43da-9bc7-1ea0aa279336n@googlegroups.com>
<abc12fce-3f84-4b4a-92fb-a715c2115d5en@googlegroups.com> <fd462aa7-b6b3-42d1-8f81-22b9326b4b8en@googlegroups.com>
<cf311410-2bb9-4a4e-b0b7-647f3a88230fn@googlegroups.com> <3457c613-1a84-4676-826f-a3fa0ab2c0fbn@googlegroups.com>
<f4c20a5f-8e0a-4db7-b955-56c97c17998bn@googlegroups.com> <de55a948-dd8d-4a0b-9025-8c3c491e8cd0n@googlegroups.com>
<64961caf-876c-4fc6-b7fd-f4903bbe0733n@googlegroups.com> <dc083541-a0c2-4e83-ba38-dee0e7036778n@googlegroups.com>
<d760773b-7579-4901-95d6-d5d1b4bbd71cn@googlegroups.com> <d5e0aeb2-cba6-4d84-a72c-506a1e4c7164n@googlegroups.com>
User-Agent: G2/1.0
MIME-Version: 1.0
Message-ID: <291e7827-e487-4b50-8c22-c127d0607d18n@googlegroups.com>
Subject: Re: The De Sitter Challenge - Has to be answered
From: trevorla...@gmail.com (Trevor Lange)
Injection-Date: Sat, 06 May 2023 18:57:11 +0000
Content-Type: text/plain; charset="UTF-8"
Content-Transfer-Encoding: quoted-printable
X-Received-Bytes: 6857
 by: Trevor Lange - Sat, 6 May 2023 18:57 UTC

On Friday, May 5, 2023 at 11:42:23 PM UTC-7, gehan.am...@gmail.com wrote:
> I do understand the need for Lorentz invariance, not about inertial of energy.

That is a self-contradictory statement. The essence of Lorentz invariance is the inertia of energy. They are essentially the same thing.

> > It is easy to determine all the objective facts involving the strike events A and B, such as the fact that tA < tB and t'A > t'B for relatively moving systems of standard inertial coordinates x,t and x',t'. Do you understand this?
>
> No. I understand that any event can be translated into another coordinate system.

That sentence makes no sense... An event has specific coordinates in terms of some specific system of coordinates, and it has other coordinates in terms of some other system of coordinates. We can determine the latter from the former by means of the transformation between those systems... that's what a coordinate transformation means. Now do you understand?

> > > I need to read more that is all.
> >
> > No, you don't need to read more, you need to *think* more, and be more honest. You are already in possession of all the information needed to understand special relativity. All you need to do is think rationally.
>
> I am thinking rationally as far as I am aware and that is true.

But that's the problem: Your brain is incapable of distinguishing between sense and nonsense, between rationality and irrationality. There may not be any solution to this. The only hope is that you aren't actually incapable, you are simply unwilling, because you value your cherished juvenile fantasies above rationality and intellectual integrity. In that case, you could conceivably reform your character, and then with your new-found intellectual integrity you could finally understand things. But if your brain is genuinely incapable of thinking rationally (as seems to be the case), there is little hope for you.

> I do not accept that that light propagates in vacuum at speed c in terms of every
> standard system of inertial coordinates.

Right, but the point is that this fact has been carefully explained to you, and you have been unable to even attempt to find any flaw in the explanation, so your denialism is irrational and dishonest. Agreed?

> Pythogoras' theorem can be easily demonstrated to be true visually.
> With Special Relativity the background assumptions are not so easily
> discerned.

They are qualitatively the very same kinds of assertions. In terms of x,y coordinates corresponding to an orthoginal grid of standard rulers, the measure s along the interval from the origin to the point at x,y satisfies s^2 = x^2 + y^2. Likewise, in terms of x,t coordinates corresponding to an orthoginal grid of standard rulers and clocks, the measure s along the interval from the origin to the event at x,t satisfies s^2 = x^2 - t^2. We can, with suitable instruments, confirm both of these propositions by direct measurements. So what exactly is the basis of your denialism?

> My practice of rationality does not allow me to accept that light is measured
> by all 'observers' as travelling at c.

Well, that is a sloppy (at best) statement. Observers can measure things in terms of any system of measurement they find convenienet, and it is not true that the speed of light is c in terms of all of them. The correct statement is that the speed of light in vacuum is c in terms of every standard system of inertial coordinates, which is a very special class of coordinate systems, with a very particular meaning. The is a logical consequence of the Pythagorean theorem up above. Also, you somehow need to get your brain to stop thinking purely in terms of light. Local Lorentz invariance is not just limited to light, it applies to everything.

> The De Sitter discussion can now be considered closed...

Well, it was closed a century ago, among rational adults. For you, however, it remains open until you can summon the intellectual integrity to set aside your cherished fantasies and start thinking rationally. Agreed?

Re: The De Sitter Challenge - Has to be answered

<175cb7b9ac247bca$1$2230647$fd385da@news.newsgroupdirect.com>

  copy mid

https://www.novabbs.com/tech/article-flat.php?id=114849&group=sci.physics.relativity#114849

  copy link   Newsgroups: sci.physics.relativity
From: Jan...@home.com (Jane)
Subject: Re: The De Sitter Challenge - Has to be answered
Newsgroups: sci.physics.relativity
References: <f8985602-5407-41f0-ae98-d2f9822bafaan@googlegroups.com> <kb9rquF9c3eU1@mid.individual.net> <d8cf070e-eecc-4d6a-8b07-761033b39d82n@googlegroups.com> <14ab18f7-e73a-4669-9711-d948e9eb9a03n@googlegroups.com> <47b7e21e-3092-48f1-b4c8-ce7b7c9c95ebn@googlegroups.com> <db6709a7-14c2-44be-a858-ddff3edb5953n@googlegroups.com> <ba2a0cf6-8427-47a1-9362-af508819f5c1n@googlegroups.com> <9af61d5f-1ba5-4d8a-b74a-8eb8af2240adn@googlegroups.com> <836c5b12-c66c-4264-9ce5-8485c1292dcdn@googlegroups.com> <c2fe1ba0-e01b-4c6a-b7ff-149d4f9045c4n@googlegroups.com> <ee14994b-3112-47d7-8a03-551c3f636987n@googlegroups.com> <55cf99e6-1675-4d83-b0b3-f5fed4cf6911n@googlegroups.com> <5c054fd2-88fc-43da-9bc7-1ea0aa279336n@googlegroups.com> <abc12fce-3f84-4b4a-92fb-a715c2115d5en@googlegroups.com> <f077e8f9-2432-46c9-a761-82f5fb925d1dn@googlegroups.com> <175c164593ff4b9e$14$406094$4bd3c1de@news.newsgroupdirect.com> <YPadnfL9vunQfMj5nZ2dnZfqlJxh4p2d@giganews.com>
User-Agent: Pan/0.144 (Time is the enemy; 28ab3ba git.gnome.org/pan2)
Mime-Version: 1.0
Content-Type: text/plain; charset=UTF-8
Content-Transfer-Encoding: 8bit
Lines: 24
Path: i2pn2.org!i2pn.org!weretis.net!feeder8.news.weretis.net!3.eu.feeder.erje.net!feeder.erje.net!border-1.nntp.ord.giganews.com!nntp.giganews.com!feeder.usenetexpress.com!tr3.iad1.usenetexpress.com!news.newsgroupdirect.com!not-for-mail
Date: Sun, 07 May 2023 01:19:28 +0000
Nntp-Posting-Date: Sun, 07 May 2023 01:19:28 +0000
X-Received-Bytes: 2104
X-Complaints-To: abuse@newsgroupdirect.com
Organization: NewsgroupDirect
Message-Id: <175cb7b9ac247bca$1$2230647$fd385da@news.newsgroupdirect.com>
 by: Jane - Sun, 7 May 2023 01:19 UTC

On Sat, 06 May 2023 00:10:37 -0500, Tom Roberts wrote:

> On 5/4/23 7:00 PM, Jane wrote:
>> Now here is NOW everywhere even if there is no way to actually prove
>> that.
>
> Once again you display your personal ignorance. That claim takes you out
> of the realm of science and into theology -- your wishes and dreams are
> no constraint on the world we inhabit, or on the theories we construct
> to describe that world. They do, however, seem to be a constraint on how
> you "think"....

Is the world you inhabit the one based on the glaring errors in Paul
Andersen's Sagnac article and Albert Michelson ridiculous moving mirror
experiment?
> Tom Roberts

--
-- lover of truth

Re: The De Sitter Challenge - Has to be answered

<c1a35879-2c5a-42d3-921b-e3bc451017b1n@googlegroups.com>

  copy mid

https://www.novabbs.com/tech/article-flat.php?id=114862&group=sci.physics.relativity#114862

  copy link   Newsgroups: sci.physics.relativity
X-Received: by 2002:a05:622a:1991:b0:3eb:14c0:b41b with SMTP id u17-20020a05622a199100b003eb14c0b41bmr2496402qtc.5.1683426124572;
Sat, 06 May 2023 19:22:04 -0700 (PDT)
X-Received: by 2002:a05:620a:4452:b0:755:e6df:f772 with SMTP id
w18-20020a05620a445200b00755e6dff772mr2365203qkp.7.1683426124306; Sat, 06 May
2023 19:22:04 -0700 (PDT)
Path: i2pn2.org!i2pn.org!weretis.net!feeder8.news.weretis.net!newsreader4.netcologne.de!news.netcologne.de!peer03.ams1!peer.ams1.xlned.com!news.xlned.com!peer01.iad!feed-me.highwinds-media.com!news.highwinds-media.com!news-out.google.com!nntp.google.com!postnews.google.com!google-groups.googlegroups.com!not-for-mail
Newsgroups: sci.physics.relativity
Date: Sat, 6 May 2023 19:22:04 -0700 (PDT)
In-Reply-To: <175cb7b9ac247bca$1$2230647$fd385da@news.newsgroupdirect.com>
Injection-Info: google-groups.googlegroups.com; posting-host=97.130.205.81; posting-account=FyvUbwkAAAARAfp2CSw2Km63SBNL9trz
NNTP-Posting-Host: 97.130.205.81
References: <f8985602-5407-41f0-ae98-d2f9822bafaan@googlegroups.com>
<kb9rquF9c3eU1@mid.individual.net> <d8cf070e-eecc-4d6a-8b07-761033b39d82n@googlegroups.com>
<14ab18f7-e73a-4669-9711-d948e9eb9a03n@googlegroups.com> <47b7e21e-3092-48f1-b4c8-ce7b7c9c95ebn@googlegroups.com>
<db6709a7-14c2-44be-a858-ddff3edb5953n@googlegroups.com> <ba2a0cf6-8427-47a1-9362-af508819f5c1n@googlegroups.com>
<9af61d5f-1ba5-4d8a-b74a-8eb8af2240adn@googlegroups.com> <836c5b12-c66c-4264-9ce5-8485c1292dcdn@googlegroups.com>
<c2fe1ba0-e01b-4c6a-b7ff-149d4f9045c4n@googlegroups.com> <ee14994b-3112-47d7-8a03-551c3f636987n@googlegroups.com>
<55cf99e6-1675-4d83-b0b3-f5fed4cf6911n@googlegroups.com> <5c054fd2-88fc-43da-9bc7-1ea0aa279336n@googlegroups.com>
<abc12fce-3f84-4b4a-92fb-a715c2115d5en@googlegroups.com> <f077e8f9-2432-46c9-a761-82f5fb925d1dn@googlegroups.com>
<175c164593ff4b9e$14$406094$4bd3c1de@news.newsgroupdirect.com>
<YPadnfL9vunQfMj5nZ2dnZfqlJxh4p2d@giganews.com> <175cb7b9ac247bca$1$2230647$fd385da@news.newsgroupdirect.com>
User-Agent: G2/1.0
MIME-Version: 1.0
Message-ID: <c1a35879-2c5a-42d3-921b-e3bc451017b1n@googlegroups.com>
Subject: Re: The De Sitter Challenge - Has to be answered
From: pnals...@gmail.com (Paul Alsing)
Injection-Date: Sun, 07 May 2023 02:22:04 +0000
Content-Type: text/plain; charset="UTF-8"
Content-Transfer-Encoding: quoted-printable
X-Received-Bytes: 3086
 by: Paul Alsing - Sun, 7 May 2023 02:22 UTC

On Saturday, May 6, 2023 at 6:21:30 PM UTC-7, Jane wrote:
> On Sat, 06 May 2023 00:10:37 -0500, Tom Roberts wrote:
>
> > On 5/4/23 7:00 PM, Jane wrote:
> >> Now here is NOW everywhere even if there is no way to actually prove
> >> that.
> >
> > Once again you display your personal ignorance. That claim takes you out
> > of the realm of science and into theology -- your wishes and dreams are
> > no constraint on the world we inhabit, or on the theories we construct
> > to describe that world. They do, however, seem to be a constraint on how
> > you "think"....

> Is the world you inhabit the one based on the glaring errors in Paul
> Andersen's Sagnac article and Albert Michelson ridiculous moving mirror
> experiment?

What errors? Be precise...

Re: The De Sitter Challenge - Has to be answered

<effee474-12e1-4ca8-a20e-a195633d0275n@googlegroups.com>

  copy mid

https://www.novabbs.com/tech/article-flat.php?id=114881&group=sci.physics.relativity#114881

  copy link   Newsgroups: sci.physics.relativity
X-Received: by 2002:a05:622a:1a17:b0:3e3:8172:ff21 with SMTP id f23-20020a05622a1a1700b003e38172ff21mr2517010qtb.8.1683432360323;
Sat, 06 May 2023 21:06:00 -0700 (PDT)
X-Received: by 2002:a05:622a:189d:b0:3bf:da0f:ed7c with SMTP id
v29-20020a05622a189d00b003bfda0fed7cmr2402756qtc.11.1683432360085; Sat, 06
May 2023 21:06:00 -0700 (PDT)
Path: i2pn2.org!i2pn.org!weretis.net!feeder6.news.weretis.net!news.misty.com!border-2.nntp.ord.giganews.com!border-1.nntp.ord.giganews.com!nntp.giganews.com!news-out.google.com!nntp.google.com!postnews.google.com!google-groups.googlegroups.com!not-for-mail
Newsgroups: sci.physics.relativity
Date: Sat, 6 May 2023 21:05:59 -0700 (PDT)
In-Reply-To: <291e7827-e487-4b50-8c22-c127d0607d18n@googlegroups.com>
Injection-Info: google-groups.googlegroups.com; posting-host=185.215.33.40; posting-account=sVBCDQoAAAADe-Ogi2R38m91EmLrcIgt
NNTP-Posting-Host: 185.215.33.40
References: <f8985602-5407-41f0-ae98-d2f9822bafaan@googlegroups.com>
<kb9fh2F7st1U1@mid.individual.net> <217730f2-8df8-444a-9dc8-91ff22fe8e0an@googlegroups.com>
<kb9rquF9c3eU1@mid.individual.net> <d8cf070e-eecc-4d6a-8b07-761033b39d82n@googlegroups.com>
<14ab18f7-e73a-4669-9711-d948e9eb9a03n@googlegroups.com> <47b7e21e-3092-48f1-b4c8-ce7b7c9c95ebn@googlegroups.com>
<db6709a7-14c2-44be-a858-ddff3edb5953n@googlegroups.com> <ba2a0cf6-8427-47a1-9362-af508819f5c1n@googlegroups.com>
<9af61d5f-1ba5-4d8a-b74a-8eb8af2240adn@googlegroups.com> <836c5b12-c66c-4264-9ce5-8485c1292dcdn@googlegroups.com>
<c2fe1ba0-e01b-4c6a-b7ff-149d4f9045c4n@googlegroups.com> <ee14994b-3112-47d7-8a03-551c3f636987n@googlegroups.com>
<55cf99e6-1675-4d83-b0b3-f5fed4cf6911n@googlegroups.com> <5c054fd2-88fc-43da-9bc7-1ea0aa279336n@googlegroups.com>
<abc12fce-3f84-4b4a-92fb-a715c2115d5en@googlegroups.com> <fd462aa7-b6b3-42d1-8f81-22b9326b4b8en@googlegroups.com>
<cf311410-2bb9-4a4e-b0b7-647f3a88230fn@googlegroups.com> <3457c613-1a84-4676-826f-a3fa0ab2c0fbn@googlegroups.com>
<f4c20a5f-8e0a-4db7-b955-56c97c17998bn@googlegroups.com> <de55a948-dd8d-4a0b-9025-8c3c491e8cd0n@googlegroups.com>
<64961caf-876c-4fc6-b7fd-f4903bbe0733n@googlegroups.com> <dc083541-a0c2-4e83-ba38-dee0e7036778n@googlegroups.com>
<d760773b-7579-4901-95d6-d5d1b4bbd71cn@googlegroups.com> <d5e0aeb2-cba6-4d84-a72c-506a1e4c7164n@googlegroups.com>
<291e7827-e487-4b50-8c22-c127d0607d18n@googlegroups.com>
User-Agent: G2/1.0
MIME-Version: 1.0
Message-ID: <effee474-12e1-4ca8-a20e-a195633d0275n@googlegroups.com>
Subject: Re: The De Sitter Challenge - Has to be answered
From: gehan.am...@gmail.com (gehan.am...@gmail.com)
Injection-Date: Sun, 07 May 2023 04:06:00 +0000
Content-Type: text/plain; charset="UTF-8"
Content-Transfer-Encoding: quoted-printable
Lines: 120
 by: gehan.am...@gmail.co - Sun, 7 May 2023 04:05 UTC

On Saturday, May 6, 2023 at 11:57:13 PM UTC+5, Trevor Lange wrote:
> On Friday, May 5, 2023 at 11:42:23 PM UTC-7, gehan.am...@gmail.com wrote:
> > I do understand the need for Lorentz invariance, not about inertial of energy.
> That is a self-contradictory statement. The essence of Lorentz invariance is the inertia of energy. They are essentially the same thing.

Quite a lot covered here. Any references to the inertia of energy, I would like to have a look at it at a reputable site.
I am not disputing this one, just need more information, I have not come across this before.

> > > It is easy to determine all the objective facts involving the strike events A and B, such as the fact that tA < tB and t'A > t'B for relatively moving systems of standard inertial coordinates x,t and x',t'. Do you understand this?
> >
> > No. I understand that any event can be translated into another coordinate system.
> That sentence makes no sense... An event has specific coordinates in terms of some specific system of coordinates, and it has other coordinates in terms of some other system of coordinates. We can determine the latter from the former by means of the transformation between those systems... that's what a coordinate transformation means. Now do you understand?

I understand that. An event has coordinates x,y,z and t in a specific system of coordinates. In another system of coordinates, it has coordinates x'y'z' and t' but t <> t' unlike in a Newtonian universe. Is this correct?

> > > > I need to read more that is all.
> > >
> > > No, you don't need to read more, you need to *think* more, and be more honest. You are already in possession of all the information needed to understand special relativity. All you need to do is think rationally.

I will be thinking more, that I promise, but it will be offline.
> >
> > I am thinking rationally as far as I am aware and that is true.
> But that's the problem: Your brain is incapable of distinguishing between sense and nonsense, between rationality and irrationality. There may not be any solution to this. The only hope is that you aren't actually incapable, you are simply unwilling, because you value your cherished juvenile fantasies above rationality and intellectual integrity. In that case, you could conceivably reform your character, and then with your new-found intellectual integrity you could finally understand things. But if your brain is genuinely incapable of thinking rationally (as seems to be the case), there is little hope for you.

There are tests for rationality. I intend to take them.

> > I do not accept that that light propagates in vacuum at speed c in terms of every
> > standard system of inertial coordinates.
> Right, but the point is that this fact has been carefully explained to you, and you have been unable to even attempt to find any flaw in the explanation, so your denialism is irrational and dishonest. Agreed?

Neither. How can a student accept the answer to a question when it does not make any sense? Put it down to lack of mathematical
skills, intelligence, but not to dishonesty. Irrationality... well maybe...it is possible, however that is another debate.

> > Pythogoras' theorem can be easily demonstrated to be true visually.
> > With Special Relativity the background assumptions are not so easily
> > discerned.
> They are qualitatively the very same kinds of assertions. In terms of x,y coordinates corresponding to an orthoginal grid of standard rulers, the measure s along the interval from the origin to the point at x,y satisfies s^2 = x^2 + y^2. Likewise, in terms of x,t coordinates corresponding to an orthoginal grid of standard rulers and clocks, the measure s along the interval from the origin to the event at x,t satisfies s^2 = x^2 - t^2. We can, with suitable instruments, confirm both of these propositions by direct measurements. So what exactly is the basis of your denialism?
>

Do you mean the Hafele- Keating experiment and others? "We can, with suitable instruments, confirm both of these propositions by direct measurements."

> > My practice of rationality does not allow me to accept that light is measured
> > by all 'observers' as travelling at c.

> Well, that is a sloppy (at best) statement. Observers can measure things in terms of any system of measurement they find convenienet, and it is not true that the speed of light is c in terms of all of them. The correct statement is that the speed of light in vacuum is c in terms of every standard system of inertial coordinates, which is a very special class of coordinate systems, with a very particular meaning. The is a logical consequence of the Pythagorean theorem up above. Also, you somehow need to get your brain to stop thinking purely in terms of light. Local Lorentz invariance is not just limited to light, it applies to everything.
>
I an accept that the speed of a bullet from a gun is the same in every inertial system of coordinates, also the speed of the growth of crystals. This is simply the first postulate. Where I run into problems is the statement that the velocity of the source relative to the detector does not affect the velocity of transmission. Hopefully, further thought with a pen and paper will clarify this for me.

"Local Lorentz invariance is not just limited to light, it applies to everything." Since everything moves at some fraction of the speed of light?

> > The De Sitter discussion can now be considered closed...
>
> Well, it was closed a century ago, among rational adults. For you, however, it remains open until you can summon the intellectual integrity to set aside your cherished fantasies and start thinking rationally. Agreed?

Not at all, I have to find away of integrating this into my view of how the universe works. Or give up my view of how the universe
works.

There is the possibility to privately and secretly set aside my 'prejudices' and look at this thing honestly, I think we can agree.
There is no risk associate with that. That is one way out. Also there has to be a way of presenting arguments more subtly, as in my thread "Challenge: prove that this invariance of the speed of light leads to logical inconsistencies"

Re: The De Sitter Challenge - Has to be answered

<64fa9c6f-17d9-4420-b068-dbf305c54ee6n@googlegroups.com>

  copy mid

https://www.novabbs.com/tech/article-flat.php?id=114905&group=sci.physics.relativity#114905

  copy link   Newsgroups: sci.physics.relativity
X-Received: by 2002:a05:622a:16:b0:3de:bafb:82b0 with SMTP id x22-20020a05622a001600b003debafb82b0mr2965087qtw.6.1683469400253;
Sun, 07 May 2023 07:23:20 -0700 (PDT)
X-Received: by 2002:a05:622a:4ce:b0:3f3:641b:85f with SMTP id
q14-20020a05622a04ce00b003f3641b085fmr2752074qtx.10.1683469400046; Sun, 07
May 2023 07:23:20 -0700 (PDT)
Path: i2pn2.org!i2pn.org!usenet.blueworldhosting.com!diablo1.usenet.blueworldhosting.com!peer01.iad!feed-me.highwinds-media.com!news.highwinds-media.com!news-out.google.com!nntp.google.com!postnews.google.com!google-groups.googlegroups.com!not-for-mail
Newsgroups: sci.physics.relativity
Date: Sun, 7 May 2023 07:23:19 -0700 (PDT)
In-Reply-To: <effee474-12e1-4ca8-a20e-a195633d0275n@googlegroups.com>
Injection-Info: google-groups.googlegroups.com; posting-host=2601:601:1700:7df0:31fa:af86:a94e:35c9;
posting-account=B2MNBQoAAADtgq_pZTEECSkLIDJGrDSJ
NNTP-Posting-Host: 2601:601:1700:7df0:31fa:af86:a94e:35c9
References: <f8985602-5407-41f0-ae98-d2f9822bafaan@googlegroups.com>
<kb9fh2F7st1U1@mid.individual.net> <217730f2-8df8-444a-9dc8-91ff22fe8e0an@googlegroups.com>
<kb9rquF9c3eU1@mid.individual.net> <d8cf070e-eecc-4d6a-8b07-761033b39d82n@googlegroups.com>
<14ab18f7-e73a-4669-9711-d948e9eb9a03n@googlegroups.com> <47b7e21e-3092-48f1-b4c8-ce7b7c9c95ebn@googlegroups.com>
<db6709a7-14c2-44be-a858-ddff3edb5953n@googlegroups.com> <ba2a0cf6-8427-47a1-9362-af508819f5c1n@googlegroups.com>
<9af61d5f-1ba5-4d8a-b74a-8eb8af2240adn@googlegroups.com> <836c5b12-c66c-4264-9ce5-8485c1292dcdn@googlegroups.com>
<c2fe1ba0-e01b-4c6a-b7ff-149d4f9045c4n@googlegroups.com> <ee14994b-3112-47d7-8a03-551c3f636987n@googlegroups.com>
<55cf99e6-1675-4d83-b0b3-f5fed4cf6911n@googlegroups.com> <5c054fd2-88fc-43da-9bc7-1ea0aa279336n@googlegroups.com>
<abc12fce-3f84-4b4a-92fb-a715c2115d5en@googlegroups.com> <fd462aa7-b6b3-42d1-8f81-22b9326b4b8en@googlegroups.com>
<cf311410-2bb9-4a4e-b0b7-647f3a88230fn@googlegroups.com> <3457c613-1a84-4676-826f-a3fa0ab2c0fbn@googlegroups.com>
<f4c20a5f-8e0a-4db7-b955-56c97c17998bn@googlegroups.com> <de55a948-dd8d-4a0b-9025-8c3c491e8cd0n@googlegroups.com>
<64961caf-876c-4fc6-b7fd-f4903bbe0733n@googlegroups.com> <dc083541-a0c2-4e83-ba38-dee0e7036778n@googlegroups.com>
<d760773b-7579-4901-95d6-d5d1b4bbd71cn@googlegroups.com> <d5e0aeb2-cba6-4d84-a72c-506a1e4c7164n@googlegroups.com>
<291e7827-e487-4b50-8c22-c127d0607d18n@googlegroups.com> <effee474-12e1-4ca8-a20e-a195633d0275n@googlegroups.com>
User-Agent: G2/1.0
MIME-Version: 1.0
Message-ID: <64fa9c6f-17d9-4420-b068-dbf305c54ee6n@googlegroups.com>
Subject: Re: The De Sitter Challenge - Has to be answered
From: trevorla...@gmail.com (Trevor Lange)
Injection-Date: Sun, 07 May 2023 14:23:20 +0000
Content-Type: text/plain; charset="UTF-8"
Content-Transfer-Encoding: quoted-printable
X-Received-Bytes: 6898
 by: Trevor Lange - Sun, 7 May 2023 14:23 UTC

On Saturday, May 6, 2023 at 9:06:01 PM UTC-7, gehan.am...@gmail.com wrote:
> I do understand the need for Lorentz invariance, not about inertial of energy.

That's a self-contradictory statement. The essence of Lorentz invariance is the inertia of energy. They are essentially the same thing.

> Any references to the inertia of energy? I have not come across this before.

You haven't? Well, you could start with one of the most famous papers ever written, by an obscure scientist named Albert Einstein in 1905, entitled "Does the Inertia of a Body Depend on it's Energy?" This is the paper generally cited for an obscure little relation, E = mc^2, that apparently in your careful and thorough studies of the theory of relativity and related literature over the past several decades you have "never come across".

> I understand that. An event has coordinates x,y,z and t in a specific system of
> [standard inertial ] coordinates. In another system of [standard inertial] coordinates,
> it has coordinates x'y'z' and t' but t <> t' unlike in a Newtonian universe. Is this correct?

No, you omitted the crucial phrase [standard inertial]. Without that qualifier, your assertion is grossly underspecified. And, just as importantly, you continue to ignore the reason that standard inertial coordinates are related by Lorentz transformations. You claim that you want to understand this, but then you ignore the explanation, so you are being dishonest, agreed?

> There are tests for rationality. I intend to take them.

You've already failed the test. See above. To remedy, start thinking rationally, e.g., when you ask a question, start paying attention to the answer..

> How can a student accept the answer to a question when it does not make any sense?

They shouldn't. But that isn't the situation in which you find yourself. The answer you've been given makes perfect sense, and you are unable to even attempt to point out any flaw in it (or anything non-sensical about it), so this has been settled. You are simply dishonestly disregarding the answer that you claim to be seeking.

> Put it down to lack of mathematical skills, intelligence, but not to dishonesty.

Well, I agree that you lack both intelligence and math skills, but the main problem is your dishonesty, as explained above.

> > They are qualitatively the very same kinds of assertions. In terms of x,y coordinates corresponding to an orthoginal grid of standard rulers, the measure s along the interval from the origin to the point at x,y satisfies s^2 = x^2 + y^2. Likewise, in terms of x,t coordinates corresponding to an orthoginal grid of standard rulers and clocks, the measure s along the interval from the origin to the event at x,t satisfies s^2 = x^2 - t^2. We can, with suitable instruments, confirm both of these propositions by direct measurements. So what exactly is the basis of your denialism?
> >
> Do you mean the Hafele- Keating experiment and others?

No, that was just a silly publicity stunt. The empirical foundations of special relativity were established in the early 1900's, but the important point is that your objection to Lorentz invariance is not fundamentally empirical, it is logical, i.e., your brain contains the false belief that there is no logical possibility for the relativistic speed composition to be correct. Witness your thread proposing to show that this proposition entails logical contradiction. But the logical way in which the speed composition works has been patiently explained to you, so your denialism now is pure dishonesty. Agreed?

> Where I run into problems is the statement that the velocity of the source relative
> to the detector does not affect the velocity of transmission.

Again, this has been thoroughly explained to you. If there was some aspect of the explanation that was unclear to you, you could simply ask for clarification. But you don't, you just ignore the aswer and run away. That is crackpot behavior. Agreed?

Re: The De Sitter Challenge - Has to be answered

<175d0dcaca432eda$53$565070$c3d349d6@news.newsgroupdirect.com>

  copy mid

https://www.novabbs.com/tech/article-flat.php?id=114983&group=sci.physics.relativity#114983

  copy link   Newsgroups: sci.physics.relativity
From: Jan...@home.com (Jane)
Subject: Re: The De Sitter Challenge - Has to be answered
Newsgroups: sci.physics.relativity
References: <f8985602-5407-41f0-ae98-d2f9822bafaan@googlegroups.com> <14ab18f7-e73a-4669-9711-d948e9eb9a03n@googlegroups.com> <47b7e21e-3092-48f1-b4c8-ce7b7c9c95ebn@googlegroups.com> <db6709a7-14c2-44be-a858-ddff3edb5953n@googlegroups.com> <ba2a0cf6-8427-47a1-9362-af508819f5c1n@googlegroups.com> <9af61d5f-1ba5-4d8a-b74a-8eb8af2240adn@googlegroups.com> <836c5b12-c66c-4264-9ce5-8485c1292dcdn@googlegroups.com> <c2fe1ba0-e01b-4c6a-b7ff-149d4f9045c4n@googlegroups.com> <ee14994b-3112-47d7-8a03-551c3f636987n@googlegroups.com> <55cf99e6-1675-4d83-b0b3-f5fed4cf6911n@googlegroups.com> <5c054fd2-88fc-43da-9bc7-1ea0aa279336n@googlegroups.com> <abc12fce-3f84-4b4a-92fb-a715c2115d5en@googlegroups.com> <f077e8f9-2432-46c9-a761-82f5fb925d1dn@googlegroups.com> <175c164593ff4b9e$14$406094$4bd3c1de@news.newsgroupdirect.com> <YPadnfL9vunQfMj5nZ2dnZfqlJxh4p2d@giganews.com> <175cb7b9ac247bca$1$2230647$fd385da@news.newsgroupdirect.com> <c1a35879-2c5a-42d3-921b-e3bc451017b1n@googlegroups.com>
User-Agent: Pan/0.144 (Time is the enemy; 28ab3ba git.gnome.org/pan2)
Mime-Version: 1.0
Content-Type: text/plain; charset=UTF-8
Content-Transfer-Encoding: 8bit
Lines: 32
Path: i2pn2.org!i2pn.org!weretis.net!feeder6.news.weretis.net!news.misty.com!border-2.nntp.ord.giganews.com!border-1.nntp.ord.giganews.com!nntp.giganews.com!feeder.usenetexpress.com!tr1.iad1.usenetexpress.com!news.newsgroupdirect.com!not-for-mail
Date: Mon, 08 May 2023 03:36:40 +0000
Nntp-Posting-Date: Mon, 08 May 2023 03:36:40 +0000
X-Received-Bytes: 2678
Organization: NewsgroupDirect
X-Complaints-To: abuse@newsgroupdirect.com
Message-Id: <175d0dcaca432eda$53$565070$c3d349d6@news.newsgroupdirect.com>
 by: Jane - Mon, 8 May 2023 03:36 UTC

On Sat, 06 May 2023 19:22:04 -0700, Paul Alsing wrote:

> On Saturday, May 6, 2023 at 6:21:30 PM UTC-7, Jane wrote:
>> On Sat, 06 May 2023 00:10:37 -0500, Tom Roberts wrote:

>> > Once again you display your personal ignorance. That claim takes you
>> > out of the realm of science and into theology -- your wishes and
>> > dreams are no constraint on the world we inhabit, or on the theories
>> > we construct to describe that world. They do, however, seem to be a
>> > constraint on how you "think"....
>
>> Is the world you inhabit the one based on the glaring errors in Paul
>> Andersen's Sagnac article and Albert Michelson ridiculous moving mirror
>> experiment?
>
> What errors? Be precise...

Paul Andersen made a thorough mess of transforming an event from an
inertial to a rotating frame and Michelson used travel time differences
rather than wave number.....both terribly obvious mistakes. Since you are
convinced you are a genius you should have no trouble discovering why.

The Michelson actually refutes Einstein...just as most interferometry
based on travel times does. Sagnac does not refute Einstein because, like
the MMX, it secretly reverts to the Newtonian explanation.

--
-- lover of truth

Re: The De Sitter Challenge - Has to be answered

<573a1666-e7ff-47a4-8576-074d4ca1c881n@googlegroups.com>

  copy mid

https://www.novabbs.com/tech/article-flat.php?id=114989&group=sci.physics.relativity#114989

  copy link   Newsgroups: sci.physics.relativity
X-Received: by 2002:a05:6214:1843:b0:61b:6a72:d74 with SMTP id d3-20020a056214184300b0061b6a720d74mr1926897qvy.4.1683520470895;
Sun, 07 May 2023 21:34:30 -0700 (PDT)
X-Received: by 2002:a05:620a:3949:b0:757:87f7:750e with SMTP id
qs9-20020a05620a394900b0075787f7750emr101210qkn.7.1683520470535; Sun, 07 May
2023 21:34:30 -0700 (PDT)
Path: i2pn2.org!i2pn.org!weretis.net!feeder8.news.weretis.net!newsreader4.netcologne.de!news.netcologne.de!peer01.ams1!peer.ams1.xlned.com!news.xlned.com!peer01.iad!feed-me.highwinds-media.com!news.highwinds-media.com!news-out.google.com!nntp.google.com!postnews.google.com!google-groups.googlegroups.com!not-for-mail
Newsgroups: sci.physics.relativity
Date: Sun, 7 May 2023 21:34:30 -0700 (PDT)
In-Reply-To: <64fa9c6f-17d9-4420-b068-dbf305c54ee6n@googlegroups.com>
Injection-Info: google-groups.googlegroups.com; posting-host=185.215.33.40; posting-account=sVBCDQoAAAADe-Ogi2R38m91EmLrcIgt
NNTP-Posting-Host: 185.215.33.40
References: <f8985602-5407-41f0-ae98-d2f9822bafaan@googlegroups.com>
<kb9fh2F7st1U1@mid.individual.net> <217730f2-8df8-444a-9dc8-91ff22fe8e0an@googlegroups.com>
<kb9rquF9c3eU1@mid.individual.net> <d8cf070e-eecc-4d6a-8b07-761033b39d82n@googlegroups.com>
<14ab18f7-e73a-4669-9711-d948e9eb9a03n@googlegroups.com> <47b7e21e-3092-48f1-b4c8-ce7b7c9c95ebn@googlegroups.com>
<db6709a7-14c2-44be-a858-ddff3edb5953n@googlegroups.com> <ba2a0cf6-8427-47a1-9362-af508819f5c1n@googlegroups.com>
<9af61d5f-1ba5-4d8a-b74a-8eb8af2240adn@googlegroups.com> <836c5b12-c66c-4264-9ce5-8485c1292dcdn@googlegroups.com>
<c2fe1ba0-e01b-4c6a-b7ff-149d4f9045c4n@googlegroups.com> <ee14994b-3112-47d7-8a03-551c3f636987n@googlegroups.com>
<55cf99e6-1675-4d83-b0b3-f5fed4cf6911n@googlegroups.com> <5c054fd2-88fc-43da-9bc7-1ea0aa279336n@googlegroups.com>
<abc12fce-3f84-4b4a-92fb-a715c2115d5en@googlegroups.com> <fd462aa7-b6b3-42d1-8f81-22b9326b4b8en@googlegroups.com>
<cf311410-2bb9-4a4e-b0b7-647f3a88230fn@googlegroups.com> <3457c613-1a84-4676-826f-a3fa0ab2c0fbn@googlegroups.com>
<f4c20a5f-8e0a-4db7-b955-56c97c17998bn@googlegroups.com> <de55a948-dd8d-4a0b-9025-8c3c491e8cd0n@googlegroups.com>
<64961caf-876c-4fc6-b7fd-f4903bbe0733n@googlegroups.com> <dc083541-a0c2-4e83-ba38-dee0e7036778n@googlegroups.com>
<d760773b-7579-4901-95d6-d5d1b4bbd71cn@googlegroups.com> <d5e0aeb2-cba6-4d84-a72c-506a1e4c7164n@googlegroups.com>
<291e7827-e487-4b50-8c22-c127d0607d18n@googlegroups.com> <effee474-12e1-4ca8-a20e-a195633d0275n@googlegroups.com>
<64fa9c6f-17d9-4420-b068-dbf305c54ee6n@googlegroups.com>
User-Agent: G2/1.0
MIME-Version: 1.0
Message-ID: <573a1666-e7ff-47a4-8576-074d4ca1c881n@googlegroups.com>
Subject: Re: The De Sitter Challenge - Has to be answered
From: gehan.am...@gmail.com (gehan.am...@gmail.com)
Injection-Date: Mon, 08 May 2023 04:34:30 +0000
Content-Type: text/plain; charset="UTF-8"
Content-Transfer-Encoding: quoted-printable
X-Received-Bytes: 8609
 by: gehan.am...@gmail.co - Mon, 8 May 2023 04:34 UTC

On Sunday, May 7, 2023 at 7:23:21 PM UTC+5, Trevor Lange wrote:
> On Saturday, May 6, 2023 at 9:06:01 PM UTC-7, gehan.am...@gmail.com wrote:
> > I do understand the need for Lorentz invariance, not about inertial of energy.
> That's a self-contradictory statement. The essence of Lorentz invariance is the inertia of energy. They are essentially the same thing.

The statement I do understand x and do not understand y is a permissible statement according to the rules of logic.
That may be incorrect but cannot be self-contradictory.
>
> > Any references to the inertia of energy? I have not come across this before.
>
> You haven't? Well, you could start with one of the most famous papers ever written, by an obscure scientist named Albert Einstein in 1905, entitled "Does the Inertia of a Body Depend on it's Energy?" This is the paper generally cited for an obscure little relation, E = mc^2, that apparently in your careful and thorough studies of the theory of relativity and related literature over the past several decades you have "never come across".

You want me to be 'honest'? well I never saw that "The essence of Lorentz invariance is the inertia of energy".
That is a honest statement agree? Oh wait a minute, you do agree.

> > I understand that. An event has coordinates x,y,z and t in a specific system of
> > [standard inertial ] coordinates. In another system of [standard inertial] coordinates,
> > it has coordinates x'y'z' and t' but t <> t' unlike in a Newtonian universe. Is this correct?
> No, you omitted the crucial phrase [standard inertial]. Without that qualifier, your assertion is grossly underspecified. And, just as importantly, you continue to ignore the reason that standard inertial coordinates are related by Lorentz transformations. You claim that you want to understand this, but then you ignore the explanation, so you are being dishonest, agreed?

No. If you claim I am being dishonest then I see no way forward.

"standard inertial coordinates are related by Lorentz transformations" yes they are, why? Because thats what they were meant to do.

> > There are tests for rationality. I intend to take them.
> You've already failed the test. See above. To remedy, start thinking rationally, e.g., when you ask a question, start paying attention to the answer..

Irrational?

> > How can a student accept the answer to a question when it does not make any sense?
> They shouldn't. But that isn't the situation in which you find yourself. The answer you've been given makes perfect sense, and you are unable to even attempt to point out any flaw in it (or anything non-sensical about it), so this has been settled. You are simply dishonestly disregarding the answer that you claim to be seeking.

Unable?

> > Put it down to lack of mathematical skills, intelligence, but not to dishonesty.
> Well, I agree that you lack both intelligence and math skills, but the main problem is your dishonesty, as explained above.

lack both intelligence and math skills, but the main problem is your dishonesty,

> > > They are qualitatively the very same kinds of assertions. In terms of x,y coordinates corresponding to an orthoginal grid of standard rulers, the measure s along the interval from the origin to the point at x,y satisfies s^2 = x^2 + y^2. Likewise, in terms of x,t coordinates corresponding to an orthoginal grid of standard rulers and clocks, the measure s along the interval from the origin to the event at x,t satisfies s^2 = x^2 - t^2. We can, with suitable instruments, confirm both of these propositions by direct measurements. So what exactly is the basis of your denialism?
> > >

I cannot see how those experiments are without flaws. I deny the validity of those experiments, you agree on Hafele Keating. What else?

> > Do you mean the Hafele- Keating experiment and others?
> No, that was just a silly publicity stunt. The empirical foundations of special relativity were established in the early 1900's, but the important point is that your objection to Lorentz invariance is not fundamentally empirical, it is logical, i.e., your brain contains the false belief that there is no logical possibility for the relativistic speed composition to be correct. Witness your thread proposing to show that this proposition entails logical contradiction. But the logical way in which the speed composition works has been patiently explained to you, so your denialism now is pure dishonesty. Agreed?

> > Where I run into problems is the statement that the velocity of the source relative
> > to the detector does not affect the velocity of transmission.
> Again, this has been thoroughly explained to you. If there was some aspect of the explanation that was unclear to you, you could simply ask for clarification. But you don't, you just ignore the aswer and run away. That is crackpot behavior. Agreed?

I will try one more time to ask you a clarification.

Let me quote Einstein, since you are unlikely to fault that. It deserves another thread:

"Any ray of light moves in the “stationary” system of co-ordinates with
the determined velocity c, whether the ray be emitted by a stationary or by a
moving body."

This means that, in a stationary system of coordinates, it does not matter, in the above statement, whether the body is
moving or stationary with respect to that system.

It means that all moving sources of light in a stationary system can be replaced, for purpose of calculation, by a stationary body.

Agreed?

Re: The De Sitter Challenge - Has to be answered

<076074c4-c9b6-4292-990f-366602b1a7f8n@googlegroups.com>

  copy mid

https://www.novabbs.com/tech/article-flat.php?id=114990&group=sci.physics.relativity#114990

  copy link   Newsgroups: sci.physics.relativity
X-Received: by 2002:a05:620a:3951:b0:74c:f9b2:47b7 with SMTP id qs17-20020a05620a395100b0074cf9b247b7mr2898089qkn.2.1683520606322;
Sun, 07 May 2023 21:36:46 -0700 (PDT)
X-Received: by 2002:a05:620a:404a:b0:74e:324:d6da with SMTP id
i10-20020a05620a404a00b0074e0324d6damr3771140qko.7.1683520606069; Sun, 07 May
2023 21:36:46 -0700 (PDT)
Path: i2pn2.org!i2pn.org!weretis.net!feeder8.news.weretis.net!newsreader4.netcologne.de!news.netcologne.de!peer02.ams1!peer.ams1.xlned.com!news.xlned.com!peer01.iad!feed-me.highwinds-media.com!news.highwinds-media.com!news-out.google.com!nntp.google.com!postnews.google.com!google-groups.googlegroups.com!not-for-mail
Newsgroups: sci.physics.relativity
Date: Sun, 7 May 2023 21:36:45 -0700 (PDT)
In-Reply-To: <175d0dcaca432eda$53$565070$c3d349d6@news.newsgroupdirect.com>
Injection-Info: google-groups.googlegroups.com; posting-host=185.215.33.40; posting-account=sVBCDQoAAAADe-Ogi2R38m91EmLrcIgt
NNTP-Posting-Host: 185.215.33.40
References: <f8985602-5407-41f0-ae98-d2f9822bafaan@googlegroups.com>
<14ab18f7-e73a-4669-9711-d948e9eb9a03n@googlegroups.com> <47b7e21e-3092-48f1-b4c8-ce7b7c9c95ebn@googlegroups.com>
<db6709a7-14c2-44be-a858-ddff3edb5953n@googlegroups.com> <ba2a0cf6-8427-47a1-9362-af508819f5c1n@googlegroups.com>
<9af61d5f-1ba5-4d8a-b74a-8eb8af2240adn@googlegroups.com> <836c5b12-c66c-4264-9ce5-8485c1292dcdn@googlegroups.com>
<c2fe1ba0-e01b-4c6a-b7ff-149d4f9045c4n@googlegroups.com> <ee14994b-3112-47d7-8a03-551c3f636987n@googlegroups.com>
<55cf99e6-1675-4d83-b0b3-f5fed4cf6911n@googlegroups.com> <5c054fd2-88fc-43da-9bc7-1ea0aa279336n@googlegroups.com>
<abc12fce-3f84-4b4a-92fb-a715c2115d5en@googlegroups.com> <f077e8f9-2432-46c9-a761-82f5fb925d1dn@googlegroups.com>
<175c164593ff4b9e$14$406094$4bd3c1de@news.newsgroupdirect.com>
<YPadnfL9vunQfMj5nZ2dnZfqlJxh4p2d@giganews.com> <175cb7b9ac247bca$1$2230647$fd385da@news.newsgroupdirect.com>
<c1a35879-2c5a-42d3-921b-e3bc451017b1n@googlegroups.com> <175d0dcaca432eda$53$565070$c3d349d6@news.newsgroupdirect.com>
User-Agent: G2/1.0
MIME-Version: 1.0
Message-ID: <076074c4-c9b6-4292-990f-366602b1a7f8n@googlegroups.com>
Subject: Re: The De Sitter Challenge - Has to be answered
From: gehan.am...@gmail.com (gehan.am...@gmail.com)
Injection-Date: Mon, 08 May 2023 04:36:46 +0000
Content-Type: text/plain; charset="UTF-8"
Content-Transfer-Encoding: quoted-printable
X-Received-Bytes: 3820
 by: gehan.am...@gmail.co - Mon, 8 May 2023 04:36 UTC

On Monday, May 8, 2023 at 8:38:15 AM UTC+5, Jane wrote:
> On Sat, 06 May 2023 19:22:04 -0700, Paul Alsing wrote:
>
> > On Saturday, May 6, 2023 at 6:21:30 PM UTC-7, Jane wrote:
> >> On Sat, 06 May 2023 00:10:37 -0500, Tom Roberts wrote:
>
> >> > Once again you display your personal ignorance. That claim takes you
> >> > out of the realm of science and into theology -- your wishes and
> >> > dreams are no constraint on the world we inhabit, or on the theories
> >> > we construct to describe that world. They do, however, seem to be a
> >> > constraint on how you "think"....
> >
> >> Is the world you inhabit the one based on the glaring errors in Paul
> >> Andersen's Sagnac article and Albert Michelson ridiculous moving mirror
> >> experiment?
> >
> > What errors? Be precise...
> Paul Andersen made a thorough mess of transforming an event from an
> inertial to a rotating frame and Michelson used travel time differences
> rather than wave number.....both terribly obvious mistakes. Since you are
> convinced you are a genius you should have no trouble discovering why.
>
> The Michelson actually refutes Einstein...just as most interferometry
> based on travel times does. Sagnac does not refute Einstein because, like
> the MMX, it secretly reverts to the Newtonian explanation.
> --
> -- lover of truth

Sagnac? Is rotation in the Sagnac some sort of absolute rotation?

Re: The De Sitter Challenge - Has to be answered

<b16eb0ce-43db-40a2-a367-c6ffd40a5fc2n@googlegroups.com>

  copy mid

https://www.novabbs.com/tech/article-flat.php?id=115001&group=sci.physics.relativity#115001

  copy link   Newsgroups: sci.physics.relativity
X-Received: by 2002:a05:622a:1b89:b0:3ef:3510:7c3f with SMTP id bp9-20020a05622a1b8900b003ef35107c3fmr4015664qtb.12.1683525708160;
Sun, 07 May 2023 23:01:48 -0700 (PDT)
X-Received: by 2002:a05:620a:45aa:b0:74a:92e:bcfb with SMTP id
bp42-20020a05620a45aa00b0074a092ebcfbmr3793881qkb.3.1683525707946; Sun, 07
May 2023 23:01:47 -0700 (PDT)
Path: i2pn2.org!i2pn.org!usenet.blueworldhosting.com!diablo1.usenet.blueworldhosting.com!peer01.iad!feed-me.highwinds-media.com!news.highwinds-media.com!news-out.google.com!nntp.google.com!postnews.google.com!google-groups.googlegroups.com!not-for-mail
Newsgroups: sci.physics.relativity
Date: Sun, 7 May 2023 23:01:47 -0700 (PDT)
In-Reply-To: <573a1666-e7ff-47a4-8576-074d4ca1c881n@googlegroups.com>
Injection-Info: google-groups.googlegroups.com; posting-host=2601:601:1700:7df0:bc8e:5e21:6e09:4930;
posting-account=B2MNBQoAAADtgq_pZTEECSkLIDJGrDSJ
NNTP-Posting-Host: 2601:601:1700:7df0:bc8e:5e21:6e09:4930
References: <f8985602-5407-41f0-ae98-d2f9822bafaan@googlegroups.com>
<kb9fh2F7st1U1@mid.individual.net> <217730f2-8df8-444a-9dc8-91ff22fe8e0an@googlegroups.com>
<kb9rquF9c3eU1@mid.individual.net> <d8cf070e-eecc-4d6a-8b07-761033b39d82n@googlegroups.com>
<14ab18f7-e73a-4669-9711-d948e9eb9a03n@googlegroups.com> <47b7e21e-3092-48f1-b4c8-ce7b7c9c95ebn@googlegroups.com>
<db6709a7-14c2-44be-a858-ddff3edb5953n@googlegroups.com> <ba2a0cf6-8427-47a1-9362-af508819f5c1n@googlegroups.com>
<9af61d5f-1ba5-4d8a-b74a-8eb8af2240adn@googlegroups.com> <836c5b12-c66c-4264-9ce5-8485c1292dcdn@googlegroups.com>
<c2fe1ba0-e01b-4c6a-b7ff-149d4f9045c4n@googlegroups.com> <ee14994b-3112-47d7-8a03-551c3f636987n@googlegroups.com>
<55cf99e6-1675-4d83-b0b3-f5fed4cf6911n@googlegroups.com> <5c054fd2-88fc-43da-9bc7-1ea0aa279336n@googlegroups.com>
<abc12fce-3f84-4b4a-92fb-a715c2115d5en@googlegroups.com> <fd462aa7-b6b3-42d1-8f81-22b9326b4b8en@googlegroups.com>
<cf311410-2bb9-4a4e-b0b7-647f3a88230fn@googlegroups.com> <3457c613-1a84-4676-826f-a3fa0ab2c0fbn@googlegroups.com>
<f4c20a5f-8e0a-4db7-b955-56c97c17998bn@googlegroups.com> <de55a948-dd8d-4a0b-9025-8c3c491e8cd0n@googlegroups.com>
<64961caf-876c-4fc6-b7fd-f4903bbe0733n@googlegroups.com> <dc083541-a0c2-4e83-ba38-dee0e7036778n@googlegroups.com>
<d760773b-7579-4901-95d6-d5d1b4bbd71cn@googlegroups.com> <d5e0aeb2-cba6-4d84-a72c-506a1e4c7164n@googlegroups.com>
<291e7827-e487-4b50-8c22-c127d0607d18n@googlegroups.com> <effee474-12e1-4ca8-a20e-a195633d0275n@googlegroups.com>
<64fa9c6f-17d9-4420-b068-dbf305c54ee6n@googlegroups.com> <573a1666-e7ff-47a4-8576-074d4ca1c881n@googlegroups.com>
User-Agent: G2/1.0
MIME-Version: 1.0
Message-ID: <b16eb0ce-43db-40a2-a367-c6ffd40a5fc2n@googlegroups.com>
Subject: Re: The De Sitter Challenge - Has to be answered
From: trevorla...@gmail.com (Trevor Lange)
Injection-Date: Mon, 08 May 2023 06:01:48 +0000
Content-Type: text/plain; charset="UTF-8"
Content-Transfer-Encoding: quoted-printable
X-Received-Bytes: 9462
 by: Trevor Lange - Mon, 8 May 2023 06:01 UTC

On Sunday, May 7, 2023 at 9:34:32 PM UTC-7, gehan.am...@gmail.com wrote:
> > > I do understand the need for Lorentz invariance, not about inertial of energy.
> >
> > That's a self-contradictory statement, because the essence of Lorentz invariance is the inertia of energy -- they are essentially the same thing.
>
> The statement I do understand x and do not understand y is a permissible statement according to the rules of logic.
> That may be incorrect but cannot be self-contradictory.

Again, your statement is self-contradictory because the essence of Lorentz invariance is the inertia of energy -- they are essentially the same thing -- so you are saying you understand x but you don't understand x. Understand?

> > > Any references to the inertia of energy? I have not come across this before.
> >
> > You haven't? Well, you could start with one of the most famous papers ever written, by an obscure scientist named Albert Einstein in 1905, entitled "Does the Inertia of a Body Depend on it's Energy?" This is the paper generally cited for an obscure little relation, E = mc^2, that apparently in your careful and thorough studies of the theory of relativity and related literature over the past several decades you have "never come across".
>
> I never saw that "The essence of Lorentz invariance is the inertia of energy".
> That is a honest statement agree?

No, you are obviously lying. You cannot possibly have never come across Einstein's 1905 paper on the inertia of energy and the relation E=mc^2. Right?

> > You claim that you want to understand this, but then you ignore the explanation, so you are being dishonest, agreed?
>
> No. If you claim I am being dishonest then I see no way forward.

Well, the rational way forward would be for you to honestly acknowledge that the logical basis and meaning of Lorentz invariance has been explained to you, and that you have not been able too point out any logical flaw in the explanation, so your belief that it entails a contradiction has been debunked, and if you still have any questions about it, go ahead and ask.

> > > How can a student accept the answer to a question when it does not make any sense?
> > They shouldn't. But that isn't the situation in which you find yourself.. The answer you've been given makes perfect sense, and you are unable to even attempt to point out any flaw in it (or anything non-sensical about it), so this has been settled. You are simply dishonestly disregarding the answer that you claim to be seeking.
>
> Unable?

Yes, if you could see a flaw in the answer to your questions you would have pointed it out... unless you are either trolling or are unable to find a flaw. I'm giving you the benefit of the doubt that you are not just trolling, so you must be unable.

> I cannot see how those experiments are without flaws.

You cannot possibly understand the empirical basis of special relativity until you understand special relativity... which you do not. For example, you currently don't even understand that Lorentz invariance corresponds to the inertia of energy, so you couldn't possibly grasp the implications of demonstrations of the inertia of energy. Your juvenile nihilism toward the experimental evidence stems from your belief that Lorentz invariance entails a logical inconsistency, so the first thing you need to do is gain an understanding of special relativity... which you refuse to do. The point of showing you the formal identity between Pythagoras' theorem and the line element of Lorentz invariance was to point out the analagous means of demonstation, but you ignored it.

They are qualitatively the very same kinds of assertions. In terms of x,y coordinates corresponding to an orthoginal grid of standard rulers, the measure s along the interval from the origin to the point at x,y satisfies s^2 = x^2 + y^2. Likewise, in terms of x,t coordinates corresponding to an orthoginal grid of standard rulers and clocks, the measure s along the interval from the origin to the event at x,t satisfies s^2 = x^2 - t^2. Your nihilism toward the latter would also, if you were being honest, require you to reject Pythagoras's theorem as well.

> I will try one more time to ask you a clarification. Let me quote Einstein,
> since you are unlikely to fault that. It deserves another thread:
> "Any ray of light moves in the “stationary” system of co-ordinates with
> the determined velocity c, whether the ray be emitted by a stationary or by a
> moving body." This means that, in a stationary system of coordinates, it
> does not matter, in the above statement, whether the body is moving or
> stationary with respect to that system.

Right!

> It means that all moving sources of light in a stationary system can be
> replaced, for purpose of calculation, by a stationary body. Agreed?

The subject isn't calculations it is objective facts, but if you are asking if there is physically no difference between sources in different states of motion, that is obviously false. The statement means precisely what it says. If there is a source of light mid-way between you and me (assume we are mutually stationary), and it is moving toward me and away from you, the speed of propagation of light pulses from that source in terms of the standard inertial coordinates in which we are at rest is c in both direction, which would also be true if the source was sitting stationary between us, or moving toward you and away from me. However, those situations are not identical in all respects. For example, if the source is moving toward me, the light moving toward me is blue-shifted in frequency and has shorter wavelength, whereas the light toward you is redshifted with longer frequency, all compared with the frequency and wavelength in terms of the standard inertial coordinates in which the source is at rest. These things depend on the relative motion of the source... but the speed of propagation does not.

Look, it's pointless for someone like you to be trying to practice the Socrating method. If you have some logical inconsistency in mind, spit it out, don't just sloooowly approach it by asking nitwit preliminary questions. You've been studying this for decades, so you ought to be able to state your case quickly and succinctly. Speed it up.

Re: The De Sitter Challenge - Has to be answered

<u3a4ld$3q3cp$1@dont-email.me>

  copy mid

https://www.novabbs.com/tech/article-flat.php?id=115002&group=sci.physics.relativity#115002

  copy link   Newsgroups: sci.physics.relativity
Path: i2pn2.org!i2pn.org!eternal-september.org!news.eternal-september.org!.POSTED!not-for-mail
From: vol...@invalid.invalid (Volney)
Newsgroups: sci.physics.relativity
Subject: Re: The De Sitter Challenge - Has to be answered
Date: Mon, 8 May 2023 02:25:54 -0400
Organization: A noiseless patient Spider
Lines: 19
Message-ID: <u3a4ld$3q3cp$1@dont-email.me>
References: <f8985602-5407-41f0-ae98-d2f9822bafaan@googlegroups.com>
<db6709a7-14c2-44be-a858-ddff3edb5953n@googlegroups.com>
<ba2a0cf6-8427-47a1-9362-af508819f5c1n@googlegroups.com>
<9af61d5f-1ba5-4d8a-b74a-8eb8af2240adn@googlegroups.com>
<836c5b12-c66c-4264-9ce5-8485c1292dcdn@googlegroups.com>
<c2fe1ba0-e01b-4c6a-b7ff-149d4f9045c4n@googlegroups.com>
<ee14994b-3112-47d7-8a03-551c3f636987n@googlegroups.com>
<55cf99e6-1675-4d83-b0b3-f5fed4cf6911n@googlegroups.com>
<5c054fd2-88fc-43da-9bc7-1ea0aa279336n@googlegroups.com>
<abc12fce-3f84-4b4a-92fb-a715c2115d5en@googlegroups.com>
<f077e8f9-2432-46c9-a761-82f5fb925d1dn@googlegroups.com>
<175c164593ff4b9e$14$406094$4bd3c1de@news.newsgroupdirect.com>
<YPadnfL9vunQfMj5nZ2dnZfqlJxh4p2d@giganews.com>
<175cb7b9ac247bca$1$2230647$fd385da@news.newsgroupdirect.com>
<c1a35879-2c5a-42d3-921b-e3bc451017b1n@googlegroups.com>
<175d0dcaca432eda$53$565070$c3d349d6@news.newsgroupdirect.com>
<076074c4-c9b6-4292-990f-366602b1a7f8n@googlegroups.com>
MIME-Version: 1.0
Content-Type: text/plain; charset=UTF-8; format=flowed
Content-Transfer-Encoding: 8bit
Injection-Date: Mon, 8 May 2023 06:25:49 -0000 (UTC)
Injection-Info: dont-email.me; posting-host="3fce3af3bc4bb56ef4273d90dc67bca1";
logging-data="4001177"; mail-complaints-to="abuse@eternal-september.org"; posting-account="U2FsdGVkX1+G+bKibsWzE+jmnaRp+ZU5"
User-Agent: Mozilla/5.0 (Windows NT 6.1; Win64; x64; rv:102.0) Gecko/20100101
Thunderbird/102.10.0
Cancel-Lock: sha1:a/Me0SE+m9RG8uiSO7IYbNgYHX4=
Content-Language: en-US
In-Reply-To: <076074c4-c9b6-4292-990f-366602b1a7f8n@googlegroups.com>
 by: Volney - Mon, 8 May 2023 06:25 UTC

On 5/8/2023 12:36 AM, gehan.am...@gmail.com wrote:
> On Monday, May 8, 2023 at 8:38:15 AM UTC+5, Jane wrote:
>> On Sat, 06 May 2023 19:22:04 -0700, Paul Alsing wrote:

>>> What errors? Be precise...
>> Paul Andersen made a thorough mess of transforming an event from an
>> inertial to a rotating frame and Michelson used travel time differences
>> rather than wave number.....both terribly obvious mistakes. Since you are
>> convinced you are a genius you should have no trouble discovering why.
>>
>> The Michelson actually refutes Einstein...just as most interferometry
>> based on travel times does. Sagnac does not refute Einstein because, like
>> the MMX, it secretly reverts to the Newtonian explanation.
>> --
>> -- lover of truth
>
> Sagnac? Is rotation in the Sagnac some sort of absolute rotation?

All rotation is absolute.

Re: The De Sitter Challenge - Has to be answered

<fvKdnYZxov6im8T5nZ2dnZfqlJxh4p2d@giganews.com>

  copy mid

https://www.novabbs.com/tech/article-flat.php?id=115040&group=sci.physics.relativity#115040

  copy link   Newsgroups: sci.physics.relativity
Path: i2pn2.org!i2pn.org!weretis.net!feeder6.news.weretis.net!news.misty.com!border-2.nntp.ord.giganews.com!nntp.giganews.com!Xl.tags.giganews.com!local-1.nntp.ord.giganews.com!news.giganews.com.POSTED!not-for-mail
NNTP-Posting-Date: Mon, 08 May 2023 14:25:03 +0000
Date: Mon, 8 May 2023 09:25:02 -0500
MIME-Version: 1.0
User-Agent: Mozilla/5.0 (Macintosh; Intel Mac OS X 10.15; rv:102.0)
Gecko/20100101 Thunderbird/102.10.1
Subject: Re: The De Sitter Challenge - Has to be answered
Content-Language: en-US
Newsgroups: sci.physics.relativity
References: <f8985602-5407-41f0-ae98-d2f9822bafaan@googlegroups.com>
<ba2a0cf6-8427-47a1-9362-af508819f5c1n@googlegroups.com>
<9af61d5f-1ba5-4d8a-b74a-8eb8af2240adn@googlegroups.com>
<836c5b12-c66c-4264-9ce5-8485c1292dcdn@googlegroups.com>
<c2fe1ba0-e01b-4c6a-b7ff-149d4f9045c4n@googlegroups.com>
<ee14994b-3112-47d7-8a03-551c3f636987n@googlegroups.com>
<55cf99e6-1675-4d83-b0b3-f5fed4cf6911n@googlegroups.com>
<5c054fd2-88fc-43da-9bc7-1ea0aa279336n@googlegroups.com>
<abc12fce-3f84-4b4a-92fb-a715c2115d5en@googlegroups.com>
<f077e8f9-2432-46c9-a761-82f5fb925d1dn@googlegroups.com>
<175c164593ff4b9e$14$406094$4bd3c1de@news.newsgroupdirect.com>
<YPadnfL9vunQfMj5nZ2dnZfqlJxh4p2d@giganews.com>
<175cb7b9ac247bca$1$2230647$fd385da@news.newsgroupdirect.com>
<c1a35879-2c5a-42d3-921b-e3bc451017b1n@googlegroups.com>
<175d0dcaca432eda$53$565070$c3d349d6@news.newsgroupdirect.com>
<076074c4-c9b6-4292-990f-366602b1a7f8n@googlegroups.com>
<u3a4ld$3q3cp$1@dont-email.me>
From: tjrobert...@sbcglobal.net (Tom Roberts)
In-Reply-To: <u3a4ld$3q3cp$1@dont-email.me>
Content-Type: text/plain; charset=UTF-8; format=flowed
Content-Transfer-Encoding: 7bit
Message-ID: <fvKdnYZxov6im8T5nZ2dnZfqlJxh4p2d@giganews.com>
Lines: 12
X-Usenet-Provider: http://www.giganews.com
X-Trace: sv3-HbThfma9WMBvCZXidKptkgselHyNGBXb88CB34pPu2iFy3JzlQDg2NR3zj2FAuEEUgic0TqQbZg3qot!UhBPPkCYcciFT5oAta6GMYYPJjiNreo/ZPYOd1PQY2fu//NziucXHhcK8Fqi1VM4AxgXouwJ8XDR!cg==
X-Complaints-To: abuse@giganews.com
X-DMCA-Notifications: http://www.giganews.com/info/dmca.html
X-Abuse-and-DMCA-Info: Please be sure to forward a copy of ALL headers
X-Abuse-and-DMCA-Info: Otherwise we will be unable to process your complaint properly
X-Postfilter: 1.3.40
 by: Tom Roberts - Mon, 8 May 2023 14:25 UTC

On 5/8/23 1:25 AM, Volney wrote:
> All rotation is absolute.

Yes and no, and the details matter.

Coordinates that are rotating relative to one (locally) inertial frame
are rotating with respect to all (locally) inertial frames -- that is an
aspect of "absolute". But the rotation rate of those coordinates has
different values in different (locally) inertial frames -- that is
inconsistent with an aspect of "absolute".

Tom Roberts

Re: The De Sitter Challenge - Has to be answered

<d83dba3d-ac00-4e7e-a3a6-4b28002d53ebn@googlegroups.com>

  copy mid

https://www.novabbs.com/tech/article-flat.php?id=115047&group=sci.physics.relativity#115047

  copy link   Newsgroups: sci.physics.relativity
X-Received: by 2002:a05:620a:1921:b0:74d:f84c:a132 with SMTP id bj33-20020a05620a192100b0074df84ca132mr3959055qkb.9.1683560283773;
Mon, 08 May 2023 08:38:03 -0700 (PDT)
X-Received: by 2002:ac8:5bc2:0:b0:3ef:2d41:3e9e with SMTP id
b2-20020ac85bc2000000b003ef2d413e9emr4618127qtb.4.1683560283520; Mon, 08 May
2023 08:38:03 -0700 (PDT)
Path: i2pn2.org!i2pn.org!weretis.net!feeder8.news.weretis.net!feeder1.feed.usenet.farm!feed.usenet.farm!peer03.ams4!peer.am4.highwinds-media.com!peer03.iad!feed-me.highwinds-media.com!news.highwinds-media.com!news-out.google.com!nntp.google.com!postnews.google.com!google-groups.googlegroups.com!not-for-mail
Newsgroups: sci.physics.relativity
Date: Mon, 8 May 2023 08:38:03 -0700 (PDT)
In-Reply-To: <u3a4ld$3q3cp$1@dont-email.me>
Injection-Info: google-groups.googlegroups.com; posting-host=185.215.33.40; posting-account=sVBCDQoAAAADe-Ogi2R38m91EmLrcIgt
NNTP-Posting-Host: 185.215.33.40
References: <f8985602-5407-41f0-ae98-d2f9822bafaan@googlegroups.com>
<db6709a7-14c2-44be-a858-ddff3edb5953n@googlegroups.com> <ba2a0cf6-8427-47a1-9362-af508819f5c1n@googlegroups.com>
<9af61d5f-1ba5-4d8a-b74a-8eb8af2240adn@googlegroups.com> <836c5b12-c66c-4264-9ce5-8485c1292dcdn@googlegroups.com>
<c2fe1ba0-e01b-4c6a-b7ff-149d4f9045c4n@googlegroups.com> <ee14994b-3112-47d7-8a03-551c3f636987n@googlegroups.com>
<55cf99e6-1675-4d83-b0b3-f5fed4cf6911n@googlegroups.com> <5c054fd2-88fc-43da-9bc7-1ea0aa279336n@googlegroups.com>
<abc12fce-3f84-4b4a-92fb-a715c2115d5en@googlegroups.com> <f077e8f9-2432-46c9-a761-82f5fb925d1dn@googlegroups.com>
<175c164593ff4b9e$14$406094$4bd3c1de@news.newsgroupdirect.com>
<YPadnfL9vunQfMj5nZ2dnZfqlJxh4p2d@giganews.com> <175cb7b9ac247bca$1$2230647$fd385da@news.newsgroupdirect.com>
<c1a35879-2c5a-42d3-921b-e3bc451017b1n@googlegroups.com> <175d0dcaca432eda$53$565070$c3d349d6@news.newsgroupdirect.com>
<076074c4-c9b6-4292-990f-366602b1a7f8n@googlegroups.com> <u3a4ld$3q3cp$1@dont-email.me>
User-Agent: G2/1.0
MIME-Version: 1.0
Message-ID: <d83dba3d-ac00-4e7e-a3a6-4b28002d53ebn@googlegroups.com>
Subject: Re: The De Sitter Challenge - Has to be answered
From: gehan.am...@gmail.com (gehan.am...@gmail.com)
Injection-Date: Mon, 08 May 2023 15:38:03 +0000
Content-Type: text/plain; charset="UTF-8"
Content-Transfer-Encoding: quoted-printable
X-Received-Bytes: 3340
 by: gehan.am...@gmail.co - Mon, 8 May 2023 15:38 UTC

On Monday, May 8, 2023 at 11:25:52 AM UTC+5, Volney wrote:
> On 5/8/2023 12:36 AM, gehan.am...@gmail.com wrote:
> > On Monday, May 8, 2023 at 8:38:15 AM UTC+5, Jane wrote:
> >> On Sat, 06 May 2023 19:22:04 -0700, Paul Alsing wrote:
>
> >>> What errors? Be precise...
> >> Paul Andersen made a thorough mess of transforming an event from an
> >> inertial to a rotating frame and Michelson used travel time differences
> >> rather than wave number.....both terribly obvious mistakes. Since you are
> >> convinced you are a genius you should have no trouble discovering why.
> >>
> >> The Michelson actually refutes Einstein...just as most interferometry
> >> based on travel times does. Sagnac does not refute Einstein because, like
> >> the MMX, it secretly reverts to the Newtonian explanation.
> >> --
> >> -- lover of truth
> >
> > Sagnac? Is rotation in the Sagnac some sort of absolute rotation?
> All rotation is absolute.

Can a gyroscope or 'inertial navigation device' detect absolute motion?

Re: The De Sitter Challenge - Has to be answered

<c9a81cb6-4f7e-42c3-b419-ab7d727f7a82n@googlegroups.com>

  copy mid

https://www.novabbs.com/tech/article-flat.php?id=115048&group=sci.physics.relativity#115048

  copy link   Newsgroups: sci.physics.relativity
X-Received: by 2002:a05:620a:4051:b0:74e:5103:2c9c with SMTP id i17-20020a05620a405100b0074e51032c9cmr4116131qko.12.1683561015121;
Mon, 08 May 2023 08:50:15 -0700 (PDT)
X-Received: by 2002:a05:620a:4484:b0:74d:ed60:be11 with SMTP id
x4-20020a05620a448400b0074ded60be11mr3978131qkp.15.1683561014761; Mon, 08 May
2023 08:50:14 -0700 (PDT)
Path: i2pn2.org!i2pn.org!weretis.net!feeder8.news.weretis.net!newsreader4.netcologne.de!news.netcologne.de!fu-berlin.de!news-out.google.com!nntp.google.com!postnews.google.com!google-groups.googlegroups.com!not-for-mail
Newsgroups: sci.physics.relativity
Date: Mon, 8 May 2023 08:50:14 -0700 (PDT)
In-Reply-To: <b16eb0ce-43db-40a2-a367-c6ffd40a5fc2n@googlegroups.com>
Injection-Info: google-groups.googlegroups.com; posting-host=185.215.33.40; posting-account=sVBCDQoAAAADe-Ogi2R38m91EmLrcIgt
NNTP-Posting-Host: 185.215.33.40
References: <f8985602-5407-41f0-ae98-d2f9822bafaan@googlegroups.com>
<kb9fh2F7st1U1@mid.individual.net> <217730f2-8df8-444a-9dc8-91ff22fe8e0an@googlegroups.com>
<kb9rquF9c3eU1@mid.individual.net> <d8cf070e-eecc-4d6a-8b07-761033b39d82n@googlegroups.com>
<14ab18f7-e73a-4669-9711-d948e9eb9a03n@googlegroups.com> <47b7e21e-3092-48f1-b4c8-ce7b7c9c95ebn@googlegroups.com>
<db6709a7-14c2-44be-a858-ddff3edb5953n@googlegroups.com> <ba2a0cf6-8427-47a1-9362-af508819f5c1n@googlegroups.com>
<9af61d5f-1ba5-4d8a-b74a-8eb8af2240adn@googlegroups.com> <836c5b12-c66c-4264-9ce5-8485c1292dcdn@googlegroups.com>
<c2fe1ba0-e01b-4c6a-b7ff-149d4f9045c4n@googlegroups.com> <ee14994b-3112-47d7-8a03-551c3f636987n@googlegroups.com>
<55cf99e6-1675-4d83-b0b3-f5fed4cf6911n@googlegroups.com> <5c054fd2-88fc-43da-9bc7-1ea0aa279336n@googlegroups.com>
<abc12fce-3f84-4b4a-92fb-a715c2115d5en@googlegroups.com> <fd462aa7-b6b3-42d1-8f81-22b9326b4b8en@googlegroups.com>
<cf311410-2bb9-4a4e-b0b7-647f3a88230fn@googlegroups.com> <3457c613-1a84-4676-826f-a3fa0ab2c0fbn@googlegroups.com>
<f4c20a5f-8e0a-4db7-b955-56c97c17998bn@googlegroups.com> <de55a948-dd8d-4a0b-9025-8c3c491e8cd0n@googlegroups.com>
<64961caf-876c-4fc6-b7fd-f4903bbe0733n@googlegroups.com> <dc083541-a0c2-4e83-ba38-dee0e7036778n@googlegroups.com>
<d760773b-7579-4901-95d6-d5d1b4bbd71cn@googlegroups.com> <d5e0aeb2-cba6-4d84-a72c-506a1e4c7164n@googlegroups.com>
<291e7827-e487-4b50-8c22-c127d0607d18n@googlegroups.com> <effee474-12e1-4ca8-a20e-a195633d0275n@googlegroups.com>
<64fa9c6f-17d9-4420-b068-dbf305c54ee6n@googlegroups.com> <573a1666-e7ff-47a4-8576-074d4ca1c881n@googlegroups.com>
<b16eb0ce-43db-40a2-a367-c6ffd40a5fc2n@googlegroups.com>
User-Agent: G2/1.0
MIME-Version: 1.0
Message-ID: <c9a81cb6-4f7e-42c3-b419-ab7d727f7a82n@googlegroups.com>
Subject: Re: The De Sitter Challenge - Has to be answered
From: gehan.am...@gmail.com (gehan.am...@gmail.com)
Injection-Date: Mon, 08 May 2023 15:50:15 +0000
Content-Type: text/plain; charset="UTF-8"
Content-Transfer-Encoding: quoted-printable
 by: gehan.am...@gmail.co - Mon, 8 May 2023 15:50 UTC

On Monday, May 8, 2023 at 11:01:49 AM UTC+5, Trevor Lange wrote:
> On Sunday, May 7, 2023 at 9:34:32 PM UTC-7, gehan.am...@gmail.com wrote:
> > > > I do understand the need for Lorentz invariance, not about inertial of energy.
> > >
> > > That's a self-contradictory statement, because the essence of Lorentz invariance is the inertia of energy -- they are essentially the same thing..
> >
> > The statement I do understand x and do not understand y is a permissible statement according to the rules of logic.
> > That may be incorrect but cannot be self-contradictory.
> Again, your statement is self-contradictory because the essence of Lorentz invariance is the inertia of energy -- they are essentially the same thing -- so you are saying you understand x but you don't understand x. Understand?
> > > > Any references to the inertia of energy? I have not come across this before.
> > >
> > > You haven't? Well, you could start with one of the most famous papers ever written, by an obscure scientist named Albert Einstein in 1905, entitled "Does the Inertia of a Body Depend on it's Energy?" This is the paper generally cited for an obscure little relation, E = mc^2, that apparently in your careful and thorough studies of the theory of relativity and related literature over the past several decades you have "never come across".
> >
> > I never saw that "The essence of Lorentz invariance is the inertia of energy".
> > That is a honest statement agree?
> No, you are obviously lying. You cannot possibly have never come across Einstein's 1905 paper on the inertia of energy and the relation E=mc^2. Right?
> > > You claim that you want to understand this, but then you ignore the explanation, so you are being dishonest, agreed?
> >
> > No. If you claim I am being dishonest then I see no way forward.
> Well, the rational way forward would be for you to honestly acknowledge that the logical basis and meaning of Lorentz invariance has been explained to you, and that you have not been able too point out any logical flaw in the explanation, so your belief that it entails a contradiction has been debunked, and if you still have any questions about it, go ahead and ask.
> > > > How can a student accept the answer to a question when it does not make any sense?
> > > They shouldn't. But that isn't the situation in which you find yourself. The answer you've been given makes perfect sense, and you are unable to even attempt to point out any flaw in it (or anything non-sensical about it), so this has been settled. You are simply dishonestly disregarding the answer that you claim to be seeking.
> >
> > Unable?
> Yes, if you could see a flaw in the answer to your questions you would have pointed it out... unless you are either trolling or are unable to find a flaw. I'm giving you the benefit of the doubt that you are not just trolling, so you must be unable.
> > I cannot see how those experiments are without flaws.
> You cannot possibly understand the empirical basis of special relativity until you understand special relativity... which you do not. For example, you currently don't even understand that Lorentz invariance corresponds to the inertia of energy, so you couldn't possibly grasp the implications of demonstrations of the inertia of energy. Your juvenile nihilism toward the experimental evidence stems from your belief that Lorentz invariance entails a logical inconsistency, so the first thing you need to do is gain an understanding of special relativity... which you refuse to do. The point of showing you the formal identity between Pythagoras' theorem and the line element of Lorentz invariance was to point out the analagous means of demonstation, but you ignored it.
>
> They are qualitatively the very same kinds of assertions. In terms of x,y coordinates corresponding to an orthoginal grid of standard rulers, the measure s along the interval from the origin to the point at x,y satisfies s^2 = x^2 + y^2. Likewise, in terms of x,t coordinates corresponding to an orthoginal grid of standard rulers and clocks, the measure s along the interval from the origin to the event at x,t satisfies s^2 = x^2 - t^2. Your nihilism toward the latter would also, if you were being honest, require you to reject Pythagoras's theorem as well.
> > I will try one more time to ask you a clarification. Let me quote Einstein,
> > since you are unlikely to fault that. It deserves another thread:
> > "Any ray of light moves in the “stationary” system of co-ordinates with
> > the determined velocity c, whether the ray be emitted by a stationary or by a
> > moving body." This means that, in a stationary system of coordinates, it
> > does not matter, in the above statement, whether the body is moving or
> > stationary with respect to that system.
> Right!
> > It means that all moving sources of light in a stationary system can be
> > replaced, for purpose of calculation, by a stationary body. Agreed?
> The subject isn't calculations it is objective facts, but if you are asking if there is physically no difference between sources in different states of motion, that is obviously false. The statement means precisely what it says. If there is a source of light mid-way between you and me (assume we are mutually stationary), and it is moving toward me and away from you, the speed of propagation of light pulses from that source in terms of the standard inertial coordinates in which we are at rest is c in both direction, which would also be true if the source was sitting stationary between us, or moving toward you and away from me. However, those situations are not identical in all respects. For example, if the source is moving toward me, the light moving toward me is blue-shifted in frequency and has shorter wavelength, whereas the light toward you is redshifted with longer frequency, all compared with the frequency and wavelength in terms of the standard inertial coordinates in which the source is at rest. These things depend on the relative motion of the source... but the speed of propagation does not.
>

According to the constancy of the speed of light postulate, yes, it does look like there is a medium at rest in our frame of reference and light moves in it like a wave. I said as if.

This is curious, because in the experiment with the lightning strikes, the source is anything but stationary in the frame of reference of the observer on the train. I have tried to point this out many time before.

Distance between observer on train and lightning strikes remains the same (source taken as stationary in observers frame so he will see lightning strikes at the same time, just like the person on the track)

*L======================[__________O__________]==================*L

===*L=====================[__________O__________]====================*L

Distance between observer on train and lightning strikes changes (source 'moving')

*L=========================[__________O__________]===============*L
*L==========================[__________O__________]=============*L=
"Source moving"

Re: The De Sitter Challenge - Has to be answered

<baf9c2bd-150d-48de-b2fc-7a871faf2ddcn@googlegroups.com>

  copy mid

https://www.novabbs.com/tech/article-flat.php?id=115052&group=sci.physics.relativity#115052

  copy link   Newsgroups: sci.physics.relativity
X-Received: by 2002:a05:6214:1774:b0:61a:2953:abc9 with SMTP id et20-20020a056214177400b0061a2953abc9mr2230904qvb.2.1683563814657;
Mon, 08 May 2023 09:36:54 -0700 (PDT)
X-Received: by 2002:ac8:5b48:0:b0:3f3:8da2:4347 with SMTP id
n8-20020ac85b48000000b003f38da24347mr2016620qtw.8.1683563814376; Mon, 08 May
2023 09:36:54 -0700 (PDT)
Path: i2pn2.org!i2pn.org!weretis.net!feeder6.news.weretis.net!usenet.blueworldhosting.com!diablo1.usenet.blueworldhosting.com!peer03.iad!feed-me.highwinds-media.com!news.highwinds-media.com!news-out.google.com!nntp.google.com!postnews.google.com!google-groups.googlegroups.com!not-for-mail
Newsgroups: sci.physics.relativity
Date: Mon, 8 May 2023 09:36:54 -0700 (PDT)
In-Reply-To: <c9a81cb6-4f7e-42c3-b419-ab7d727f7a82n@googlegroups.com>
Injection-Info: google-groups.googlegroups.com; posting-host=2800:150:125:111d:155:41b6:f0b8:8939;
posting-account=KA67VQoAAAABNtRUVf2Wh-jHtkEfmXxT
NNTP-Posting-Host: 2800:150:125:111d:155:41b6:f0b8:8939
References: <f8985602-5407-41f0-ae98-d2f9822bafaan@googlegroups.com>
<kb9fh2F7st1U1@mid.individual.net> <217730f2-8df8-444a-9dc8-91ff22fe8e0an@googlegroups.com>
<kb9rquF9c3eU1@mid.individual.net> <d8cf070e-eecc-4d6a-8b07-761033b39d82n@googlegroups.com>
<14ab18f7-e73a-4669-9711-d948e9eb9a03n@googlegroups.com> <47b7e21e-3092-48f1-b4c8-ce7b7c9c95ebn@googlegroups.com>
<db6709a7-14c2-44be-a858-ddff3edb5953n@googlegroups.com> <ba2a0cf6-8427-47a1-9362-af508819f5c1n@googlegroups.com>
<9af61d5f-1ba5-4d8a-b74a-8eb8af2240adn@googlegroups.com> <836c5b12-c66c-4264-9ce5-8485c1292dcdn@googlegroups.com>
<c2fe1ba0-e01b-4c6a-b7ff-149d4f9045c4n@googlegroups.com> <ee14994b-3112-47d7-8a03-551c3f636987n@googlegroups.com>
<55cf99e6-1675-4d83-b0b3-f5fed4cf6911n@googlegroups.com> <5c054fd2-88fc-43da-9bc7-1ea0aa279336n@googlegroups.com>
<abc12fce-3f84-4b4a-92fb-a715c2115d5en@googlegroups.com> <fd462aa7-b6b3-42d1-8f81-22b9326b4b8en@googlegroups.com>
<cf311410-2bb9-4a4e-b0b7-647f3a88230fn@googlegroups.com> <3457c613-1a84-4676-826f-a3fa0ab2c0fbn@googlegroups.com>
<f4c20a5f-8e0a-4db7-b955-56c97c17998bn@googlegroups.com> <de55a948-dd8d-4a0b-9025-8c3c491e8cd0n@googlegroups.com>
<64961caf-876c-4fc6-b7fd-f4903bbe0733n@googlegroups.com> <dc083541-a0c2-4e83-ba38-dee0e7036778n@googlegroups.com>
<d760773b-7579-4901-95d6-d5d1b4bbd71cn@googlegroups.com> <d5e0aeb2-cba6-4d84-a72c-506a1e4c7164n@googlegroups.com>
<291e7827-e487-4b50-8c22-c127d0607d18n@googlegroups.com> <effee474-12e1-4ca8-a20e-a195633d0275n@googlegroups.com>
<64fa9c6f-17d9-4420-b068-dbf305c54ee6n@googlegroups.com> <573a1666-e7ff-47a4-8576-074d4ca1c881n@googlegroups.com>
<b16eb0ce-43db-40a2-a367-c6ffd40a5fc2n@googlegroups.com> <c9a81cb6-4f7e-42c3-b419-ab7d727f7a82n@googlegroups.com>
User-Agent: G2/1.0
MIME-Version: 1.0
Message-ID: <baf9c2bd-150d-48de-b2fc-7a871faf2ddcn@googlegroups.com>
Subject: Re: The De Sitter Challenge - Has to be answered
From: mri...@ing.puc.cl (Paparios)
Injection-Date: Mon, 08 May 2023 16:36:54 +0000
Content-Type: text/plain; charset="UTF-8"
Content-Transfer-Encoding: quoted-printable
X-Received-Bytes: 6759
 by: Paparios - Mon, 8 May 2023 16:36 UTC

El lunes, 8 de mayo de 2023 a las 11:50:16 UTC-4, gehan.am...@gmail.com escribió:
> On Monday, May 8, 2023 at 11:01:49 AM UTC+5, Trevor Lange wrote:

> > The subject isn't calculations it is objective facts, but if you are asking if there is physically no difference between sources in different states of motion, that is obviously false. The statement means precisely what it says. If there is a source of light mid-way between you and me (assume we are mutually stationary), and it is moving toward me and away from you, the speed of propagation of light pulses from that source in terms of the standard inertial coordinates in which we are at rest is c in both direction, which would also be true if the source was sitting stationary between us, or moving toward you and away from me. However, those situations are not identical in all respects. For example, if the source is moving toward me, the light moving toward me is blue-shifted in frequency and has shorter wavelength, whereas the light toward you is redshifted with longer frequency, all compared with the frequency and wavelength in terms of the standard inertial coordinates in which the source is at rest. These things depend on the relative motion of the source... but the speed of propagation does not.
> >

> According to the constancy of the speed of light postulate, yes, it does look like there is a medium at rest in our frame of reference and light moves in it like a wave. I said as if.
>
> This is curious, because in the experiment with the lightning strikes, the source is anything but stationary in the frame of reference of the observer on the train. I have tried to point this out many time before.
>

This is clearly wrong (see https://www.bartleby.com/lit-hub/relativity-the-special-and-general-theory/ix-the-relativity-of-simultaneity/).
In figure 1, Einstein clearly sets the experiment: "Are two events (e.g. the two strokes of lightning A and B) which are simultaneous with reference to the railway embankment".
Therefore the light source is not moving at all.

>
> Distance between observer on train and lightning strikes remains the same (source taken as stationary in observers frame so he will see lightning strikes at the same time, just like the person on the track)
>

Nonsense!!. In figure 1, at time t=0, A and B are fixed points on the embankment. Furthermore observers M (on the embankment) and M' inside the train coincide at t=0. Of course, for t>0, observer M' moves to the right of observer M, with speed v.

The light, coming from the A strike, takes a finite time to arrive to observer M (say ta seconds with ta>0). It is obvious that M' is not longer at the middle of AB, but it has moved to the right of observer M. Therefore, the light from A arrives first to M and later to M'.
The same way, light coming from point B first encounters M' and later arrives to M.

> *L======================[__________O__________]==================*L
>
> ===*L=====================[__________O__________]====================*L
>
> Distance between observer on train and lightning strikes changes (source 'moving')
>
> *L=========================[__________O__________]===============*L=
>
> *L==========================[__________O__________]=============*L==
>
> "Source moving"

Re: The De Sitter Challenge - Has to be answered

<09606849-1a08-4c97-b30c-ec3ee6e80520n@googlegroups.com>

  copy mid

https://www.novabbs.com/tech/article-flat.php?id=115054&group=sci.physics.relativity#115054

  copy link   Newsgroups: sci.physics.relativity
X-Received: by 2002:a05:622a:1981:b0:3ef:6035:465 with SMTP id u1-20020a05622a198100b003ef60350465mr4304469qtc.8.1683564177069;
Mon, 08 May 2023 09:42:57 -0700 (PDT)
X-Received: by 2002:a05:622a:1791:b0:3f0:a400:712e with SMTP id
s17-20020a05622a179100b003f0a400712emr4152865qtk.10.1683564176774; Mon, 08
May 2023 09:42:56 -0700 (PDT)
Path: i2pn2.org!i2pn.org!weretis.net!feeder8.news.weretis.net!feeder1.feed.usenet.farm!feed.usenet.farm!peer01.ams4!peer.am4.highwinds-media.com!peer03.iad!feed-me.highwinds-media.com!news.highwinds-media.com!news-out.google.com!nntp.google.com!postnews.google.com!google-groups.googlegroups.com!not-for-mail
Newsgroups: sci.physics.relativity
Date: Mon, 8 May 2023 09:42:56 -0700 (PDT)
In-Reply-To: <d83dba3d-ac00-4e7e-a3a6-4b28002d53ebn@googlegroups.com>
Injection-Info: google-groups.googlegroups.com; posting-host=2600:1700:79b3:2800:210c:ed56:b90a:ef0b;
posting-account=vma-PgoAAABrctSmMdefNKZ-c5S8buvP
NNTP-Posting-Host: 2600:1700:79b3:2800:210c:ed56:b90a:ef0b
References: <f8985602-5407-41f0-ae98-d2f9822bafaan@googlegroups.com>
<db6709a7-14c2-44be-a858-ddff3edb5953n@googlegroups.com> <ba2a0cf6-8427-47a1-9362-af508819f5c1n@googlegroups.com>
<9af61d5f-1ba5-4d8a-b74a-8eb8af2240adn@googlegroups.com> <836c5b12-c66c-4264-9ce5-8485c1292dcdn@googlegroups.com>
<c2fe1ba0-e01b-4c6a-b7ff-149d4f9045c4n@googlegroups.com> <ee14994b-3112-47d7-8a03-551c3f636987n@googlegroups.com>
<55cf99e6-1675-4d83-b0b3-f5fed4cf6911n@googlegroups.com> <5c054fd2-88fc-43da-9bc7-1ea0aa279336n@googlegroups.com>
<abc12fce-3f84-4b4a-92fb-a715c2115d5en@googlegroups.com> <f077e8f9-2432-46c9-a761-82f5fb925d1dn@googlegroups.com>
<175c164593ff4b9e$14$406094$4bd3c1de@news.newsgroupdirect.com>
<YPadnfL9vunQfMj5nZ2dnZfqlJxh4p2d@giganews.com> <175cb7b9ac247bca$1$2230647$fd385da@news.newsgroupdirect.com>
<c1a35879-2c5a-42d3-921b-e3bc451017b1n@googlegroups.com> <175d0dcaca432eda$53$565070$c3d349d6@news.newsgroupdirect.com>
<076074c4-c9b6-4292-990f-366602b1a7f8n@googlegroups.com> <u3a4ld$3q3cp$1@dont-email.me>
<d83dba3d-ac00-4e7e-a3a6-4b28002d53ebn@googlegroups.com>
User-Agent: G2/1.0
MIME-Version: 1.0
Message-ID: <09606849-1a08-4c97-b30c-ec3ee6e80520n@googlegroups.com>
Subject: Re: The De Sitter Challenge - Has to be answered
From: eggy2001...@gmail.com (Dono.)
Injection-Date: Mon, 08 May 2023 16:42:57 +0000
Content-Type: text/plain; charset="UTF-8"
Content-Transfer-Encoding: quoted-printable
X-Received-Bytes: 3740
 by: Dono. - Mon, 8 May 2023 16:42 UTC

On Monday, May 8, 2023 at 8:38:05 AM UTC-7, gehan.am...@gmail.com wrote:
> On Monday, May 8, 2023 at 11:25:52 AM UTC+5, Volney wrote:
> > On 5/8/2023 12:36 AM, gehan.am...@gmail.com wrote:
> > > On Monday, May 8, 2023 at 8:38:15 AM UTC+5, Jane wrote:
> > >> On Sat, 06 May 2023 19:22:04 -0700, Paul Alsing wrote:
> >
> > >>> What errors? Be precise...
> > >> Paul Andersen made a thorough mess of transforming an event from an
> > >> inertial to a rotating frame and Michelson used travel time differences
> > >> rather than wave number.....both terribly obvious mistakes. Since you are
> > >> convinced you are a genius you should have no trouble discovering why.
> > >>
> > >> The Michelson actually refutes Einstein...just as most interferometry
> > >> based on travel times does. Sagnac does not refute Einstein because, like
> > >> the MMX, it secretly reverts to the Newtonian explanation.
> > >> --
> > >> -- lover of truth
> > >
> > > Sagnac? Is rotation in the Sagnac some sort of absolute rotation?
> > All rotation is absolute.
> Can a gyroscope or 'inertial navigation device' detect absolute motion?

Gyroscopes detect rotation. ONLY.
Your hope of "detecting absolute motion" is a crank's errand that you have been engaged on for years.

Re: The De Sitter Challenge - Has to be answered

<61f079d0-9daf-412e-a9b6-40e4bf7a3773n@googlegroups.com>

  copy mid

https://www.novabbs.com/tech/article-flat.php?id=115058&group=sci.physics.relativity#115058

  copy link   Newsgroups: sci.physics.relativity
X-Received: by 2002:a05:622a:11c3:b0:3ef:1c64:a9ff with SMTP id n3-20020a05622a11c300b003ef1c64a9ffmr4584818qtk.10.1683570504839;
Mon, 08 May 2023 11:28:24 -0700 (PDT)
X-Received: by 2002:a05:620a:4003:b0:755:e6df:f777 with SMTP id
h3-20020a05620a400300b00755e6dff777mr4108740qko.6.1683570504605; Mon, 08 May
2023 11:28:24 -0700 (PDT)
Path: i2pn2.org!i2pn.org!weretis.net!feeder6.news.weretis.net!news.misty.com!border-2.nntp.ord.giganews.com!border-1.nntp.ord.giganews.com!nntp.giganews.com!news-out.google.com!nntp.google.com!postnews.google.com!google-groups.googlegroups.com!not-for-mail
Newsgroups: sci.physics.relativity
Date: Mon, 8 May 2023 11:28:24 -0700 (PDT)
In-Reply-To: <baf9c2bd-150d-48de-b2fc-7a871faf2ddcn@googlegroups.com>
Injection-Info: google-groups.googlegroups.com; posting-host=89.206.14.16; posting-account=I3DWzAoAAACOmZUdDcZ-C0PqAZGVsbW0
NNTP-Posting-Host: 89.206.14.16
References: <f8985602-5407-41f0-ae98-d2f9822bafaan@googlegroups.com>
<kb9fh2F7st1U1@mid.individual.net> <217730f2-8df8-444a-9dc8-91ff22fe8e0an@googlegroups.com>
<kb9rquF9c3eU1@mid.individual.net> <d8cf070e-eecc-4d6a-8b07-761033b39d82n@googlegroups.com>
<14ab18f7-e73a-4669-9711-d948e9eb9a03n@googlegroups.com> <47b7e21e-3092-48f1-b4c8-ce7b7c9c95ebn@googlegroups.com>
<db6709a7-14c2-44be-a858-ddff3edb5953n@googlegroups.com> <ba2a0cf6-8427-47a1-9362-af508819f5c1n@googlegroups.com>
<9af61d5f-1ba5-4d8a-b74a-8eb8af2240adn@googlegroups.com> <836c5b12-c66c-4264-9ce5-8485c1292dcdn@googlegroups.com>
<c2fe1ba0-e01b-4c6a-b7ff-149d4f9045c4n@googlegroups.com> <ee14994b-3112-47d7-8a03-551c3f636987n@googlegroups.com>
<55cf99e6-1675-4d83-b0b3-f5fed4cf6911n@googlegroups.com> <5c054fd2-88fc-43da-9bc7-1ea0aa279336n@googlegroups.com>
<abc12fce-3f84-4b4a-92fb-a715c2115d5en@googlegroups.com> <fd462aa7-b6b3-42d1-8f81-22b9326b4b8en@googlegroups.com>
<cf311410-2bb9-4a4e-b0b7-647f3a88230fn@googlegroups.com> <3457c613-1a84-4676-826f-a3fa0ab2c0fbn@googlegroups.com>
<f4c20a5f-8e0a-4db7-b955-56c97c17998bn@googlegroups.com> <de55a948-dd8d-4a0b-9025-8c3c491e8cd0n@googlegroups.com>
<64961caf-876c-4fc6-b7fd-f4903bbe0733n@googlegroups.com> <dc083541-a0c2-4e83-ba38-dee0e7036778n@googlegroups.com>
<d760773b-7579-4901-95d6-d5d1b4bbd71cn@googlegroups.com> <d5e0aeb2-cba6-4d84-a72c-506a1e4c7164n@googlegroups.com>
<291e7827-e487-4b50-8c22-c127d0607d18n@googlegroups.com> <effee474-12e1-4ca8-a20e-a195633d0275n@googlegroups.com>
<64fa9c6f-17d9-4420-b068-dbf305c54ee6n@googlegroups.com> <573a1666-e7ff-47a4-8576-074d4ca1c881n@googlegroups.com>
<b16eb0ce-43db-40a2-a367-c6ffd40a5fc2n@googlegroups.com> <c9a81cb6-4f7e-42c3-b419-ab7d727f7a82n@googlegroups.com>
<baf9c2bd-150d-48de-b2fc-7a871faf2ddcn@googlegroups.com>
User-Agent: G2/1.0
MIME-Version: 1.0
Message-ID: <61f079d0-9daf-412e-a9b6-40e4bf7a3773n@googlegroups.com>
Subject: Re: The De Sitter Challenge - Has to be answered
From: maluwozn...@gmail.com (Maciej Wozniak)
Injection-Date: Mon, 08 May 2023 18:28:24 +0000
Content-Type: text/plain; charset="UTF-8"
Content-Transfer-Encoding: quoted-printable
Lines: 56
 by: Maciej Wozniak - Mon, 8 May 2023 18:28 UTC

On Monday, 8 May 2023 at 18:36:55 UTC+2, Paparios wrote:
> El lunes, 8 de mayo de 2023 a las 11:50:16 UTC-4, gehan.am...@gmail.com escribió:
> > On Monday, May 8, 2023 at 11:01:49 AM UTC+5, Trevor Lange wrote:
>
> > > The subject isn't calculations it is objective facts, but if you are asking if there is physically no difference between sources in different states of motion, that is obviously false. The statement means precisely what it says. If there is a source of light mid-way between you and me (assume we are mutually stationary), and it is moving toward me and away from you, the speed of propagation of light pulses from that source in terms of the standard inertial coordinates in which we are at rest is c in both direction, which would also be true if the source was sitting stationary between us, or moving toward you and away from me. However, those situations are not identical in all respects. For example, if the source is moving toward me, the light moving toward me is blue-shifted in frequency and has shorter wavelength, whereas the light toward you is redshifted with longer frequency, all compared with the frequency and wavelength in terms of the standard inertial coordinates in which the source is at rest. These things depend on the relative motion of the source... but the speed of propagation does not.
> > >
>
> > According to the constancy of the speed of light postulate, yes, it does look like there is a medium at rest in our frame of reference and light moves in it like a wave. I said as if.
> >
> > This is curious, because in the experiment with the lightning strikes, the source is anything but stationary in the frame of reference of the observer on the train. I have tried to point this out many time before.
> >
> This is clearly wrong (see https://www.bartleby.com/lit-hub/relativity-the-special-and-general-theory/ix-the-relativity-of-simultaneity/).
> In figure 1, Einstein clearly sets the experiment: "Are two events (e.g. the two strokes of lightning A and B) which are simultaneous with reference to the railway embankment".
> Therefore the light source is not moving at all.
> >
> > Distance between observer on train and lightning strikes remains the same (source taken as stationary in observers frame so he will see lightning strikes at the same time, just like the person on the track)
> >
> Nonsense!!. In figure 1, at time t=0, A and B are fixed points on the embankment. Furthermore observers M (on the embankment) and M' inside the train coincide at t=0. Of course, for t>0, observer M' moves to the right of observer M, with speed v.
>
> The light, coming from the A strike, takes a finite time to arrive to observer M (say ta seconds with ta>0). It is obvious that M' is not longer at the middle of AB, but it has moved to the right of observer M. Therefore, the light from A arrives first to M and later to M'.
> The same way, light coming from point B first encounters M' and later arrives to M.

And in the meantime in the real world, forbidden by
your bunch of idiots improper clocks keep measuring
t'=t in improper seconds.

Re: The De Sitter Challenge - Has to be answered

<f62d8918-ff14-4190-9f52-02716852bfa3n@googlegroups.com>

  copy mid

https://www.novabbs.com/tech/article-flat.php?id=115069&group=sci.physics.relativity#115069

  copy link   Newsgroups: sci.physics.relativity
X-Received: by 2002:a05:620a:2815:b0:753:2f4b:f5b6 with SMTP id f21-20020a05620a281500b007532f4bf5b6mr2661098qkp.3.1683574289481;
Mon, 08 May 2023 12:31:29 -0700 (PDT)
X-Received: by 2002:a05:622a:138a:b0:3f3:7869:d2d2 with SMTP id
o10-20020a05622a138a00b003f37869d2d2mr4911828qtk.12.1683574289239; Mon, 08
May 2023 12:31:29 -0700 (PDT)
Path: i2pn2.org!i2pn.org!weretis.net!feeder8.news.weretis.net!feeder1.feed.usenet.farm!feed.usenet.farm!peer03.ams4!peer.am4.highwinds-media.com!peer03.iad!feed-me.highwinds-media.com!news.highwinds-media.com!news-out.google.com!nntp.google.com!postnews.google.com!google-groups.googlegroups.com!not-for-mail
Newsgroups: sci.physics.relativity
Date: Mon, 8 May 2023 12:31:29 -0700 (PDT)
In-Reply-To: <c9a81cb6-4f7e-42c3-b419-ab7d727f7a82n@googlegroups.com>
Injection-Info: google-groups.googlegroups.com; posting-host=2601:601:1700:7df0:908b:42d6:4f83:75e3;
posting-account=B2MNBQoAAADtgq_pZTEECSkLIDJGrDSJ
NNTP-Posting-Host: 2601:601:1700:7df0:908b:42d6:4f83:75e3
References: <f8985602-5407-41f0-ae98-d2f9822bafaan@googlegroups.com>
<kb9fh2F7st1U1@mid.individual.net> <217730f2-8df8-444a-9dc8-91ff22fe8e0an@googlegroups.com>
<kb9rquF9c3eU1@mid.individual.net> <d8cf070e-eecc-4d6a-8b07-761033b39d82n@googlegroups.com>
<14ab18f7-e73a-4669-9711-d948e9eb9a03n@googlegroups.com> <47b7e21e-3092-48f1-b4c8-ce7b7c9c95ebn@googlegroups.com>
<db6709a7-14c2-44be-a858-ddff3edb5953n@googlegroups.com> <ba2a0cf6-8427-47a1-9362-af508819f5c1n@googlegroups.com>
<9af61d5f-1ba5-4d8a-b74a-8eb8af2240adn@googlegroups.com> <836c5b12-c66c-4264-9ce5-8485c1292dcdn@googlegroups.com>
<c2fe1ba0-e01b-4c6a-b7ff-149d4f9045c4n@googlegroups.com> <ee14994b-3112-47d7-8a03-551c3f636987n@googlegroups.com>
<55cf99e6-1675-4d83-b0b3-f5fed4cf6911n@googlegroups.com> <5c054fd2-88fc-43da-9bc7-1ea0aa279336n@googlegroups.com>
<abc12fce-3f84-4b4a-92fb-a715c2115d5en@googlegroups.com> <fd462aa7-b6b3-42d1-8f81-22b9326b4b8en@googlegroups.com>
<cf311410-2bb9-4a4e-b0b7-647f3a88230fn@googlegroups.com> <3457c613-1a84-4676-826f-a3fa0ab2c0fbn@googlegroups.com>
<f4c20a5f-8e0a-4db7-b955-56c97c17998bn@googlegroups.com> <de55a948-dd8d-4a0b-9025-8c3c491e8cd0n@googlegroups.com>
<64961caf-876c-4fc6-b7fd-f4903bbe0733n@googlegroups.com> <dc083541-a0c2-4e83-ba38-dee0e7036778n@googlegroups.com>
<d760773b-7579-4901-95d6-d5d1b4bbd71cn@googlegroups.com> <d5e0aeb2-cba6-4d84-a72c-506a1e4c7164n@googlegroups.com>
<291e7827-e487-4b50-8c22-c127d0607d18n@googlegroups.com> <effee474-12e1-4ca8-a20e-a195633d0275n@googlegroups.com>
<64fa9c6f-17d9-4420-b068-dbf305c54ee6n@googlegroups.com> <573a1666-e7ff-47a4-8576-074d4ca1c881n@googlegroups.com>
<b16eb0ce-43db-40a2-a367-c6ffd40a5fc2n@googlegroups.com> <c9a81cb6-4f7e-42c3-b419-ab7d727f7a82n@googlegroups.com>
User-Agent: G2/1.0
MIME-Version: 1.0
Message-ID: <f62d8918-ff14-4190-9f52-02716852bfa3n@googlegroups.com>
Subject: Re: The De Sitter Challenge - Has to be answered
From: trevorla...@gmail.com (Trevor Lange)
Injection-Date: Mon, 08 May 2023 19:31:29 +0000
Content-Type: text/plain; charset="UTF-8"
Content-Transfer-Encoding: quoted-printable
X-Received-Bytes: 4577
 by: Trevor Lange - Mon, 8 May 2023 19:31 UTC

On Monday, May 8, 2023 at 8:50:16 AM UTC-7, gehan.am...@gmail.com wrote:
> According to the constancy of the speed of light postulate, yes, it does look
> like there is a medium at rest in our frame of reference and light moves in it
> like a wave. I said as if.

The incompleteness of your statement makes it wrong. According to special relativity, light propagates (in vacuum) at speed c in terms of *every* standard system of inertial coordinates (i.e., coordinates in terms of which the equations of physics hold good in their simple homogeneous and isotropic form). So, this does NOT "look like" there is a classical medium at rest in some particular frame of reference, because in that case the speed would be c only in terms of one system, not all.

> This is curious, because in the experiment with the lightning strikes, the source
> is anything but stationary in the frame of reference of the observer on the train.

Again, light propagates at the speed c in terms of every standard system of inertial coordinates, regardless of the speed of the source, so there is nothing "curious" about this. It is perfectly logically consistent, and in agreement with the explicit explanations, and with the train and lightning example, and every other example and situation. If there is something unclear about this, just ask. (Note: Your drawings are inherently inadequate to represent the two-dimensional spatio-temporal relations of the situation.)

Re: The De Sitter Challenge - Has to be answered

<u3bptc$3vt76$1@dont-email.me>

  copy mid

https://www.novabbs.com/tech/article-flat.php?id=115083&group=sci.physics.relativity#115083

  copy link   Newsgroups: sci.physics.relativity
Path: i2pn2.org!i2pn.org!eternal-september.org!news.eternal-september.org!.POSTED!not-for-mail
From: vol...@invalid.invalid (Volney)
Newsgroups: sci.physics.relativity
Subject: Re: The De Sitter Challenge - Has to be answered
Date: Mon, 8 May 2023 17:34:41 -0400
Organization: A noiseless patient Spider
Lines: 25
Message-ID: <u3bptc$3vt76$1@dont-email.me>
References: <f8985602-5407-41f0-ae98-d2f9822bafaan@googlegroups.com>
<9af61d5f-1ba5-4d8a-b74a-8eb8af2240adn@googlegroups.com>
<836c5b12-c66c-4264-9ce5-8485c1292dcdn@googlegroups.com>
<c2fe1ba0-e01b-4c6a-b7ff-149d4f9045c4n@googlegroups.com>
<ee14994b-3112-47d7-8a03-551c3f636987n@googlegroups.com>
<55cf99e6-1675-4d83-b0b3-f5fed4cf6911n@googlegroups.com>
<5c054fd2-88fc-43da-9bc7-1ea0aa279336n@googlegroups.com>
<abc12fce-3f84-4b4a-92fb-a715c2115d5en@googlegroups.com>
<f077e8f9-2432-46c9-a761-82f5fb925d1dn@googlegroups.com>
<175c164593ff4b9e$14$406094$4bd3c1de@news.newsgroupdirect.com>
<YPadnfL9vunQfMj5nZ2dnZfqlJxh4p2d@giganews.com>
<175cb7b9ac247bca$1$2230647$fd385da@news.newsgroupdirect.com>
<c1a35879-2c5a-42d3-921b-e3bc451017b1n@googlegroups.com>
<175d0dcaca432eda$53$565070$c3d349d6@news.newsgroupdirect.com>
<076074c4-c9b6-4292-990f-366602b1a7f8n@googlegroups.com>
<u3a4ld$3q3cp$1@dont-email.me>
<d83dba3d-ac00-4e7e-a3a6-4b28002d53ebn@googlegroups.com>
MIME-Version: 1.0
Content-Type: text/plain; charset=UTF-8; format=flowed
Content-Transfer-Encoding: 8bit
Injection-Date: Mon, 8 May 2023 21:34:36 -0000 (UTC)
Injection-Info: dont-email.me; posting-host="3fce3af3bc4bb56ef4273d90dc67bca1";
logging-data="4191462"; mail-complaints-to="abuse@eternal-september.org"; posting-account="U2FsdGVkX19cUra3oU5uFmeHahWR6xzm"
User-Agent: Mozilla/5.0 (Windows NT 6.1; Win64; x64; rv:102.0) Gecko/20100101
Thunderbird/102.10.0
Cancel-Lock: sha1:B9qFdo2RBaa5tzTmDbsxVIo9SI8=
In-Reply-To: <d83dba3d-ac00-4e7e-a3a6-4b28002d53ebn@googlegroups.com>
Content-Language: en-US
 by: Volney - Mon, 8 May 2023 21:34 UTC

On 5/8/2023 11:38 AM, gehan.am...@gmail.com wrote:
> On Monday, May 8, 2023 at 11:25:52 AM UTC+5, Volney wrote:
>> On 5/8/2023 12:36 AM, gehan.am...@gmail.com wrote:
>>> On Monday, May 8, 2023 at 8:38:15 AM UTC+5, Jane wrote:
>>>> On Sat, 06 May 2023 19:22:04 -0700, Paul Alsing wrote:
>>
>>>>> What errors? Be precise...
>>>> Paul Andersen made a thorough mess of transforming an event from an
>>>> inertial to a rotating frame and Michelson used travel time differences
>>>> rather than wave number.....both terribly obvious mistakes. Since you are
>>>> convinced you are a genius you should have no trouble discovering why.
>>>>
>>>> The Michelson actually refutes Einstein...just as most interferometry
>>>> based on travel times does. Sagnac does not refute Einstein because, like
>>>> the MMX, it secretly reverts to the Newtonian explanation.
>>>> --
>>>> -- lover of truth
>>>
>>> Sagnac? Is rotation in the Sagnac some sort of absolute rotation?
>> All rotation is absolute.
>
> Can a gyroscope or 'inertial navigation device' detect absolute motion?

No, gyroscopes detect rotation. There is no absolute motion.

Re: The De Sitter Challenge - Has to be answered

<6b10ff12-d1d5-41e1-9584-7d31ea6d80dcn@googlegroups.com>

  copy mid

https://www.novabbs.com/tech/article-flat.php?id=115095&group=sci.physics.relativity#115095

  copy link   Newsgroups: sci.physics.relativity
X-Received: by 2002:a05:620a:1729:b0:757:8a20:d19b with SMTP id az41-20020a05620a172900b007578a20d19bmr749459qkb.14.1683588301786;
Mon, 08 May 2023 16:25:01 -0700 (PDT)
X-Received: by 2002:a05:622a:1a06:b0:3f0:a9a0:45e3 with SMTP id
f6-20020a05622a1a0600b003f0a9a045e3mr5079352qtb.12.1683588301624; Mon, 08 May
2023 16:25:01 -0700 (PDT)
Path: i2pn2.org!i2pn.org!weretis.net!feeder6.news.weretis.net!1.us.feeder.erje.net!feeder.erje.net!border-1.nntp.ord.giganews.com!nntp.giganews.com!news-out.google.com!nntp.google.com!postnews.google.com!google-groups.googlegroups.com!not-for-mail
Newsgroups: sci.physics.relativity
Date: Mon, 8 May 2023 16:25:01 -0700 (PDT)
In-Reply-To: <473e2f7e-f35d-4738-a2ba-346f2b454828n@googlegroups.com>
Injection-Info: google-groups.googlegroups.com; posting-host=2601:601:1700:7df0:44b9:d4eb:d0fb:5f61;
posting-account=B2MNBQoAAADtgq_pZTEECSkLIDJGrDSJ
NNTP-Posting-Host: 2601:601:1700:7df0:44b9:d4eb:d0fb:5f61
References: <f8985602-5407-41f0-ae98-d2f9822bafaan@googlegroups.com>
<kb9fh2F7st1U1@mid.individual.net> <217730f2-8df8-444a-9dc8-91ff22fe8e0an@googlegroups.com>
<kb9rquF9c3eU1@mid.individual.net> <d8cf070e-eecc-4d6a-8b07-761033b39d82n@googlegroups.com>
<14ab18f7-e73a-4669-9711-d948e9eb9a03n@googlegroups.com> <47b7e21e-3092-48f1-b4c8-ce7b7c9c95ebn@googlegroups.com>
<db6709a7-14c2-44be-a858-ddff3edb5953n@googlegroups.com> <ba2a0cf6-8427-47a1-9362-af508819f5c1n@googlegroups.com>
<9af61d5f-1ba5-4d8a-b74a-8eb8af2240adn@googlegroups.com> <836c5b12-c66c-4264-9ce5-8485c1292dcdn@googlegroups.com>
<c2fe1ba0-e01b-4c6a-b7ff-149d4f9045c4n@googlegroups.com> <ee14994b-3112-47d7-8a03-551c3f636987n@googlegroups.com>
<55cf99e6-1675-4d83-b0b3-f5fed4cf6911n@googlegroups.com> <5c054fd2-88fc-43da-9bc7-1ea0aa279336n@googlegroups.com>
<abc12fce-3f84-4b4a-92fb-a715c2115d5en@googlegroups.com> <fd462aa7-b6b3-42d1-8f81-22b9326b4b8en@googlegroups.com>
<cf311410-2bb9-4a4e-b0b7-647f3a88230fn@googlegroups.com> <3457c613-1a84-4676-826f-a3fa0ab2c0fbn@googlegroups.com>
<f4c20a5f-8e0a-4db7-b955-56c97c17998bn@googlegroups.com> <de55a948-dd8d-4a0b-9025-8c3c491e8cd0n@googlegroups.com>
<64961caf-876c-4fc6-b7fd-f4903bbe0733n@googlegroups.com> <dc083541-a0c2-4e83-ba38-dee0e7036778n@googlegroups.com>
<d760773b-7579-4901-95d6-d5d1b4bbd71cn@googlegroups.com> <d5e0aeb2-cba6-4d84-a72c-506a1e4c7164n@googlegroups.com>
<291e7827-e487-4b50-8c22-c127d0607d18n@googlegroups.com> <effee474-12e1-4ca8-a20e-a195633d0275n@googlegroups.com>
<64fa9c6f-17d9-4420-b068-dbf305c54ee6n@googlegroups.com> <573a1666-e7ff-47a4-8576-074d4ca1c881n@googlegroups.com>
<b16eb0ce-43db-40a2-a367-c6ffd40a5fc2n@googlegroups.com> <c9a81cb6-4f7e-42c3-b419-ab7d727f7a82n@googlegroups.com>
<f62d8918-ff14-4190-9f52-02716852bfa3n@googlegroups.com> <473e2f7e-f35d-4738-a2ba-346f2b454828n@googlegroups.com>
User-Agent: G2/1.0
MIME-Version: 1.0
Message-ID: <6b10ff12-d1d5-41e1-9584-7d31ea6d80dcn@googlegroups.com>
Subject: Re: The De Sitter Challenge - Has to be answered
From: trevorla...@gmail.com (Trevor Lange)
Injection-Date: Mon, 08 May 2023 23:25:01 +0000
Content-Type: text/plain; charset="UTF-8"
Content-Transfer-Encoding: quoted-printable
Lines: 24
 by: Trevor Lange - Mon, 8 May 2023 23:25 UTC

On Monday, May 8, 2023 at 3:02:38 PM UTC-7, Prokaryotic Capase Homolog wrote:
> > > This is curious, because in the experiment with the lightning strikes, the source
> > > is anything but stationary in the frame of reference of the observer on the train.
> >
> > Again, light propagates at the speed c in terms of every standard system of inertial coordinates, regardless of the speed of the source, so there is nothing "curious" about this. It is perfectly logically consistent, and in agreement with the explicit explanations, and with the train and lightning example, and every other example and situation. If there is something unclear about this, just ask.
>
> Gehan is thinking "I can EASILY tell by measuring Doppler shift ..."

This was explained previously. Again, the speed of the source is irrelevant, as is the fact that the frequency and wavelength of the pulses in terms of a given system of coordinates may be different. The discussion in the booklet relies on nothing but the fact that light propagates at the speed c in terms of every standard system of inertial coordinates, regardless of the speed of the source. Thus there is nothing "curious" about this. If gehan thinks there is something curious about it, he needs to articulate what that is. So far, he has failed to state what he thinks is curious (let alone logically inconsistent) about it. And he never will [prediction].

Re: The De Sitter Challenge - Has to be answered

<71a9e44d-8651-4cd2-84c5-85c337cf4b5en@googlegroups.com>

  copy mid

https://www.novabbs.com/tech/article-flat.php?id=115123&group=sci.physics.relativity#115123

  copy link   Newsgroups: sci.physics.relativity
X-Received: by 2002:a05:620a:19a7:b0:74f:b51b:3de4 with SMTP id bm39-20020a05620a19a700b0074fb51b3de4mr4059101qkb.5.1683607005616;
Mon, 08 May 2023 21:36:45 -0700 (PDT)
X-Received: by 2002:a05:620a:4052:b0:74e:a87:be3 with SMTP id
i18-20020a05620a405200b0074e0a870be3mr4902513qko.6.1683607005408; Mon, 08 May
2023 21:36:45 -0700 (PDT)
Path: i2pn2.org!i2pn.org!weretis.net!feeder6.news.weretis.net!news.misty.com!border-2.nntp.ord.giganews.com!nntp.giganews.com!news-out.google.com!nntp.google.com!postnews.google.com!google-groups.googlegroups.com!not-for-mail
Newsgroups: sci.physics.relativity
Date: Mon, 8 May 2023 21:36:45 -0700 (PDT)
In-Reply-To: <1b111c21-29dc-490b-8bd0-ba5768086853n@googlegroups.com>
Injection-Info: google-groups.googlegroups.com; posting-host=2601:601:1700:7df0:44b9:d4eb:d0fb:5f61;
posting-account=B2MNBQoAAADtgq_pZTEECSkLIDJGrDSJ
NNTP-Posting-Host: 2601:601:1700:7df0:44b9:d4eb:d0fb:5f61
References: <f8985602-5407-41f0-ae98-d2f9822bafaan@googlegroups.com>
<kb9fh2F7st1U1@mid.individual.net> <217730f2-8df8-444a-9dc8-91ff22fe8e0an@googlegroups.com>
<kb9rquF9c3eU1@mid.individual.net> <d8cf070e-eecc-4d6a-8b07-761033b39d82n@googlegroups.com>
<14ab18f7-e73a-4669-9711-d948e9eb9a03n@googlegroups.com> <47b7e21e-3092-48f1-b4c8-ce7b7c9c95ebn@googlegroups.com>
<db6709a7-14c2-44be-a858-ddff3edb5953n@googlegroups.com> <ba2a0cf6-8427-47a1-9362-af508819f5c1n@googlegroups.com>
<9af61d5f-1ba5-4d8a-b74a-8eb8af2240adn@googlegroups.com> <836c5b12-c66c-4264-9ce5-8485c1292dcdn@googlegroups.com>
<c2fe1ba0-e01b-4c6a-b7ff-149d4f9045c4n@googlegroups.com> <ee14994b-3112-47d7-8a03-551c3f636987n@googlegroups.com>
<55cf99e6-1675-4d83-b0b3-f5fed4cf6911n@googlegroups.com> <5c054fd2-88fc-43da-9bc7-1ea0aa279336n@googlegroups.com>
<abc12fce-3f84-4b4a-92fb-a715c2115d5en@googlegroups.com> <fd462aa7-b6b3-42d1-8f81-22b9326b4b8en@googlegroups.com>
<cf311410-2bb9-4a4e-b0b7-647f3a88230fn@googlegroups.com> <3457c613-1a84-4676-826f-a3fa0ab2c0fbn@googlegroups.com>
<f4c20a5f-8e0a-4db7-b955-56c97c17998bn@googlegroups.com> <de55a948-dd8d-4a0b-9025-8c3c491e8cd0n@googlegroups.com>
<64961caf-876c-4fc6-b7fd-f4903bbe0733n@googlegroups.com> <dc083541-a0c2-4e83-ba38-dee0e7036778n@googlegroups.com>
<d760773b-7579-4901-95d6-d5d1b4bbd71cn@googlegroups.com> <d5e0aeb2-cba6-4d84-a72c-506a1e4c7164n@googlegroups.com>
<291e7827-e487-4b50-8c22-c127d0607d18n@googlegroups.com> <effee474-12e1-4ca8-a20e-a195633d0275n@googlegroups.com>
<64fa9c6f-17d9-4420-b068-dbf305c54ee6n@googlegroups.com> <573a1666-e7ff-47a4-8576-074d4ca1c881n@googlegroups.com>
<b16eb0ce-43db-40a2-a367-c6ffd40a5fc2n@googlegroups.com> <c9a81cb6-4f7e-42c3-b419-ab7d727f7a82n@googlegroups.com>
<f62d8918-ff14-4190-9f52-02716852bfa3n@googlegroups.com> <473e2f7e-f35d-4738-a2ba-346f2b454828n@googlegroups.com>
<6b10ff12-d1d5-41e1-9584-7d31ea6d80dcn@googlegroups.com> <1b111c21-29dc-490b-8bd0-ba5768086853n@googlegroups.com>
User-Agent: G2/1.0
MIME-Version: 1.0
Message-ID: <71a9e44d-8651-4cd2-84c5-85c337cf4b5en@googlegroups.com>
Subject: Re: The De Sitter Challenge - Has to be answered
From: trevorla...@gmail.com (Trevor Lange)
Injection-Date: Tue, 09 May 2023 04:36:45 +0000
Content-Type: text/plain; charset="UTF-8"
Content-Transfer-Encoding: quoted-printable
Lines: 23
 by: Trevor Lange - Tue, 9 May 2023 04:36 UTC

On Monday, May 8, 2023 at 8:45:25 PM UTC-7, Prokaryotic Capase Homolog wrote:
> > If gehan thinks there is something curious about it, he needs to articulate what that is. So far, he has failed to state what he thinks is curious (let alone logically inconsistent) about it. And he never will [prediction].
>
> To gehan, it must be perfectly obvious to the train observer
> that he is moving with respect to the lightning bolts...

Again, that is irrelevant. The flashes propagate at the speed c in terms of the standard inertial coordinates in which the train and the embankment are at rest, regardless of the state of motion of the sources.

> To gehan, therefore, the entire argument leading to relativity of
> simultaneity is based on false premises.

He has not made that claim to me, because that non-sequitur was anticipated and pre-emptively disposed of in our discussion, before he even had the chance to raise it. Again, the motion of the sources of the flashes in terms of the standard inertial coordinates in which the train or the embankment are at rest is entirely irrelevant. Dwelling on such irrelevancies, rather than simply pointing out that they are irrelevant, gives the impression that you yourself don't realize they are irrelevant.

Re: The De Sitter Challenge - Has to be answered

<c1e6b21c-c62b-43a6-9501-b579ff158918n@googlegroups.com>

  copy mid

https://www.novabbs.com/tech/article-flat.php?id=115125&group=sci.physics.relativity#115125

  copy link   Newsgroups: sci.physics.relativity
X-Received: by 2002:a05:6214:bc5:b0:61b:5b56:69ce with SMTP id ff5-20020a0562140bc500b0061b5b5669cemr2730450qvb.4.1683608315148;
Mon, 08 May 2023 21:58:35 -0700 (PDT)
X-Received: by 2002:ad4:4f33:0:b0:621:3617:caa3 with SMTP id
fc19-20020ad44f33000000b006213617caa3mr210114qvb.9.1683608314802; Mon, 08 May
2023 21:58:34 -0700 (PDT)
Path: i2pn2.org!i2pn.org!weretis.net!feeder6.news.weretis.net!usenet.blueworldhosting.com!diablo1.usenet.blueworldhosting.com!peer01.iad!feed-me.highwinds-media.com!news.highwinds-media.com!news-out.google.com!nntp.google.com!postnews.google.com!google-groups.googlegroups.com!not-for-mail
Newsgroups: sci.physics.relativity
Date: Mon, 8 May 2023 21:58:34 -0700 (PDT)
In-Reply-To: <baf9c2bd-150d-48de-b2fc-7a871faf2ddcn@googlegroups.com>
Injection-Info: google-groups.googlegroups.com; posting-host=185.215.33.40; posting-account=sVBCDQoAAAADe-Ogi2R38m91EmLrcIgt
NNTP-Posting-Host: 185.215.33.40
References: <f8985602-5407-41f0-ae98-d2f9822bafaan@googlegroups.com>
<kb9fh2F7st1U1@mid.individual.net> <217730f2-8df8-444a-9dc8-91ff22fe8e0an@googlegroups.com>
<kb9rquF9c3eU1@mid.individual.net> <d8cf070e-eecc-4d6a-8b07-761033b39d82n@googlegroups.com>
<14ab18f7-e73a-4669-9711-d948e9eb9a03n@googlegroups.com> <47b7e21e-3092-48f1-b4c8-ce7b7c9c95ebn@googlegroups.com>
<db6709a7-14c2-44be-a858-ddff3edb5953n@googlegroups.com> <ba2a0cf6-8427-47a1-9362-af508819f5c1n@googlegroups.com>
<9af61d5f-1ba5-4d8a-b74a-8eb8af2240adn@googlegroups.com> <836c5b12-c66c-4264-9ce5-8485c1292dcdn@googlegroups.com>
<c2fe1ba0-e01b-4c6a-b7ff-149d4f9045c4n@googlegroups.com> <ee14994b-3112-47d7-8a03-551c3f636987n@googlegroups.com>
<55cf99e6-1675-4d83-b0b3-f5fed4cf6911n@googlegroups.com> <5c054fd2-88fc-43da-9bc7-1ea0aa279336n@googlegroups.com>
<abc12fce-3f84-4b4a-92fb-a715c2115d5en@googlegroups.com> <fd462aa7-b6b3-42d1-8f81-22b9326b4b8en@googlegroups.com>
<cf311410-2bb9-4a4e-b0b7-647f3a88230fn@googlegroups.com> <3457c613-1a84-4676-826f-a3fa0ab2c0fbn@googlegroups.com>
<f4c20a5f-8e0a-4db7-b955-56c97c17998bn@googlegroups.com> <de55a948-dd8d-4a0b-9025-8c3c491e8cd0n@googlegroups.com>
<64961caf-876c-4fc6-b7fd-f4903bbe0733n@googlegroups.com> <dc083541-a0c2-4e83-ba38-dee0e7036778n@googlegroups.com>
<d760773b-7579-4901-95d6-d5d1b4bbd71cn@googlegroups.com> <d5e0aeb2-cba6-4d84-a72c-506a1e4c7164n@googlegroups.com>
<291e7827-e487-4b50-8c22-c127d0607d18n@googlegroups.com> <effee474-12e1-4ca8-a20e-a195633d0275n@googlegroups.com>
<64fa9c6f-17d9-4420-b068-dbf305c54ee6n@googlegroups.com> <573a1666-e7ff-47a4-8576-074d4ca1c881n@googlegroups.com>
<b16eb0ce-43db-40a2-a367-c6ffd40a5fc2n@googlegroups.com> <c9a81cb6-4f7e-42c3-b419-ab7d727f7a82n@googlegroups.com>
<baf9c2bd-150d-48de-b2fc-7a871faf2ddcn@googlegroups.com>
User-Agent: G2/1.0
MIME-Version: 1.0
Message-ID: <c1e6b21c-c62b-43a6-9501-b579ff158918n@googlegroups.com>
Subject: Re: The De Sitter Challenge - Has to be answered
From: gehan.am...@gmail.com (gehan.am...@gmail.com)
Injection-Date: Tue, 09 May 2023 04:58:35 +0000
Content-Type: text/plain; charset="UTF-8"
Content-Transfer-Encoding: quoted-printable
X-Received-Bytes: 6755
 by: gehan.am...@gmail.co - Tue, 9 May 2023 04:58 UTC

On Monday, May 8, 2023 at 9:36:55 PM UTC+5, Paparios wrote:
> El lunes, 8 de mayo de 2023 a las 11:50:16 UTC-4, gehan.am...@gmail.com escribió:
> > On Monday, May 8, 2023 at 11:01:49 AM UTC+5, Trevor Lange wrote:
>
> > > The subject isn't calculations it is objective facts, but if you are asking if there is physically no difference between sources in different states of motion, that is obviously false. The statement means precisely what it says. If there is a source of light mid-way between you and me (assume we are mutually stationary), and it is moving toward me and away from you, the speed of propagation of light pulses from that source in terms of the standard inertial coordinates in which we are at rest is c in both direction, which would also be true if the source was sitting stationary between us, or moving toward you and away from me. However, those situations are not identical in all respects. For example, if the source is moving toward me, the light moving toward me is blue-shifted in frequency and has shorter wavelength, whereas the light toward you is redshifted with longer frequency, all compared with the frequency and wavelength in terms of the standard inertial coordinates in which the source is at rest. These things depend on the relative motion of the source... but the speed of propagation does not.
> > >
>
> > According to the constancy of the speed of light postulate, yes, it does look like there is a medium at rest in our frame of reference and light moves in it like a wave. I said as if.
> >
> > This is curious, because in the experiment with the lightning strikes, the source is anything but stationary in the frame of reference of the observer on the train. I have tried to point this out many time before.
> >
> This is clearly wrong (see https://www.bartleby.com/lit-hub/relativity-the-special-and-general-theory/ix-the-relativity-of-simultaneity/).
> In figure 1, Einstein clearly sets the experiment: "Are two events (e.g. the two strokes of lightning A and B) which are simultaneous with reference to the railway embankment".
> Therefore the light source is not moving at all.
> >
> > Distance between observer on train and lightning strikes remains the same (source taken as stationary in observers frame so he will see lightning strikes at the same time, just like the person on the track)
> >
> Nonsense!!. In figure 1, at time t=0, A and B are fixed points on the embankment. Furthermore observers M (on the embankment) and M' inside the train coincide at t=0. Of course, for t>0, observer M' moves to the right of observer M, with speed v.

Agreed so far.
>
> The light, coming from the A strike, takes a finite time to arrive to observer M (say ta seconds with ta>0). It is obvious that M' is not longer at the middle of AB, but it has moved to the right of observer M. Therefore, the light from A arrives first to M and later to M'.

A strike is a light source. B strike is a light source.
M' is moving relative to A also relative to B.

This also means that the light source A and B are moving relative to M'

But we have seen that the light is emitted at c regardless of the 'state of motion of the light source'

So an equivalent experiment is where A and B are not moving relative to M'

Therefore M' will see the lightning strikes simultaneously.

Further proof: Suppose the light source (the superheated luminous air from the lightning path) was moving at the same velocity of M' subsequent to the strikes? Suppose the light source was moving at the speed of sound?

Irrespective of the motion of the source.


tech / sci.physics.relativity / Re: The De Sitter Challenge - Has to be answered

Pages:1234
server_pubkey.txt

rocksolid light 0.9.81
clearnet tor